lsat india 2014 sample question paper 4

45
8/13/2019 LSAT India 2014 Sample Question Paper 4 http://slidepdf.com/reader/full/lsat-india-2014-sample-question-paper-4 1/45 THE OFFICIAL LSAT—INDIA Form U -4LSI063 Actua l 2012 LSAT—India LSAT —India : All You Need Is Reason LSAC.org Sam ple Question Paper No 1

Upload: aglasem

Post on 04-Jun-2018

285 views

Category:

Documents


3 download

TRANSCRIPT

Page 1: LSAT India 2014 Sample Question Paper 4

8/13/2019 LSAT India 2014 Sample Question Paper 4

http://slidepdf.com/reader/full/lsat-india-2014-sample-question-paper-4 1/45

THE OFFICIAL LSAT—INDIA™

Form U -4LSI063

Actua l 2012 LSAT—India

LSAT —India : All You Need Is Reason ™

LSAC.org

Sam ple Question Paper No 1

Page 2: LSAT India 2014 Sample Question Paper 4

8/13/2019 LSAT India 2014 Sample Question Paper 4

http://slidepdf.com/reader/full/lsat-india-2014-sample-question-paper-4 2/45

• Introduction to the Law School Admission Test—India.................................................... ..................................... ...................1

• Scoring ..................................... ..................................... ...................................... ..................................... ................................1

• The Question Types ................................... ..................................... ..................................... ...................................... .............1• Reading Comprehension Questions .................................... ..................................... ...................................... ...................1• Analytical Reasoning Questions ..................................... ..................................... ..................................... ..........................2• Logical Reasoning Questions .................................. ...................................... ..................................... ................................3

• Taking the PrepTest Under Simulated Conditions............ ..................................... ..................................... ..........................4

• Answer Sheet ...................................................................................................................................................... .........................5

• The PrepTest . ...................................................................................................................................................... .........................7

• Answer Key.... ...................................................................................................................................................... .......................42

TABLE OF CONTENTS

The Law School Ad mission Council (LSAC) is a nonprofit corporation whose members are more than 200 law schools in the UnitedStates, Canada, an d Australia. Headquartered in Newtown, PA, USA, the Council was founded in 1947 to facilitate the law schooladmission process. The Council has grown to provide numerous products and services to law schools and to more th an 85,000 lawschool applicants e ach year.

All law schools app roved by the American Bar Association (ABA) are LSAC members. Canadian law schools recognize d by aprovincial or territo rial law society or government agency are also members. Accredited law schools outside of the Un ited Statesand Canada are eli gible for membership at the discretion of the LSAC Board of Trustees; Melbourne Law School, the University of Melbourne is the fi rst LSAC-member law school outside of North America.

LSAC does not eng age in assessing an applicant’s chances for admission to any law school; all admission decisions ar e made byindividual law schools.

Law School Admission Test—India, LSAT—India : All You Need Is Reason, and Law School Admission Council are trademarks of the LawSchool Admission Council, Inc.

©2012 by Law School Admission Council, Inc.

All rights reserved. No part of this work, including information, data, or other portions of the work published in electronic form, maybe reproduced or transmitted in any form or by any means, electronic or mechanical, including photocopying, recording, or by anyinformation storage and retrieval system, without permission of the publisher. For information, write: Communications, Law SchoolAdmission Council, 662 Penn Street, PO Box 40, Newtown PA 18940-0040, USA.

Page 3: LSAT India 2014 Sample Question Paper 4

8/13/2019 LSAT India 2014 Sample Question Paper 4

http://slidepdf.com/reader/full/lsat-india-2014-sample-question-paper-4 3/45

1

INTRODUCTION TO THE LAW SCHOOL ADMISSION TEST—INDIA

The LSAT—India is a test of acquired, high-level reading,informal reason ing, and deductive reasoning skills,developed spec ically for use by law schools in India.Although mode led on the Law School Admission Test(LSAT), it is adapted to the specic needs of Indianlegal education .

Critical-thinking skills are key to success in the practiceof law througho ut the world. The education of successfullawyers begins with assessing critical thinking skills duringthe law school a dmission process.

This PrepTest is a valuable tool for preparing for theLSAT—India. It is the actual 2012 LSAT–India. It consists of four, 35-minute sections of multiple-choice questions—one Analytical R easoning section, one ReadingComprehension section, and two Logical Reasoningsections. You ca n use this practice test most effectively bytaking it under t imed conditions as outlined in “Taking thePrepTest Under Simulated Conditions” on page 4 and onthe reverse side of the sample answer sheet.

We wish you g reat success with the test and your pursuitof a law degree.

SCORINGCredit is given f or each question a test taker answerscorrectly, and al l questions count equally. There is nonegative markin g or penalty for guessing, so a candidateshould answer e ach and every question. Test scores arereported on a p ercentile basis, comparing eachcandidate’s per formance to that of the others within hisor her candidat e group (Five-Year Integrated LL.B.Programme or Two-Year LL.M./ Three-Year LL.B.Programme). Sc ores for one candidate group cannot becompared to th ose for the other candidate group sincethey are based on group performance. So, for example, an

undergraduate candidate earning an LSAT—India score of 82.5 has performed better on the test than 82.5 percent of the total undergraduate candidate pool. This score doesnot indicate what the candidate’s standing would be withinthe post-undergraduate candidate pool. Note also thatthis score does not mean that the candidate answered 82.5percent of the LSAT—India questions correctly. Thus,LSAT—India scores tell law schools the relative strength of

the critical-thinking skills measured by the test for eachcandidate in comparison to the others in h is or hercandidate pool.

THE QUESTION TYPES

The multiple-choice questions that make u p the LSAT—India reect a broad range of academic di sciplines and areintended to give no advantage to candida tes from aparticular academic background. The LSA T—India doesnot include questions requiring the maste ry of any specicdiscipline or set of facts. For example, it d oes not test acandidate’s knowledge of history, political theory, or evengeneral knowledge. Rather, it is a test of i mportant critical-thinking skills that a student has acquired over his or hereducational lifetime. Thus, the LSAT—Indi a is differentfrom other legal-education admission test s used in India. Itmeasures a different set of skills and, even for thoseadmission tests that do partially address c ritical thinking,it measures those skills in different ways.

The four sections of the test contain thre e differentquestion types. The following material pres ents a general

discussion of the nature of each question ty pe and somestrategies that can be used in answering th em.

Reading Comprehension Questions

The purpose of LSAT—India Reading Com prehensionquestions is to measure the ability to read , withunderstanding and insight, examples of le ngthy andcomplex materials similar to those commo nly encounteredin law school. The Reading Comprehensio n section of theLSAT—India contains four sets of reading questions, eachset consisting of a selection of reading ma terial followedby ve to eight questions. The reading selection in three of

the four sets consists of a single reading passage; theother set generally contains two related shorter passages.Sets with two passages are a variant of ReadingComprehension called Comparative Reading.

Reading selections for reading comprehension questionsare drawn from subjects such as the humanities, the socialsciences, the biological and physical sciences, and issuesrelated to the law. Reading comprehension questions

LSAT—INDIA : ALL YOU NEED IS REASON

The LSAT—India is a test of reasoning and reading skills, not a test to see whether you happened to have memorizedthe right facts or equations. You can also be assured that each LSAT—India question will have a single answer that isclearly best. Before you ever see the questions, each is subjected to exacting reviews by at least 10 professionals withadvanced degrees in elds such as logic, English, and linguistics.

Page 4: LSAT India 2014 Sample Question Paper 4

8/13/2019 LSAT India 2014 Sample Question Paper 4

http://slidepdf.com/reader/full/lsat-india-2014-sample-question-paper-4 4/45

2

require you to read carefully and accurately, to determinethe relationships among the various parts of the readingselection, and to draw reasonable inferences from thematerial in the selection. The questions may ask about thefollowing characteristics of a passage or pair of passages:

• the main idea or primary purpose;

• the meaning or purpose of words or phrases used;

• information explicitly stated;

• information or ideas that can be inferred;

• the organizatio n or structure;

• the application of information in a passage to a newcontext; and

• the author’s att itude as it is revealed in the tone of apassage or the language used.

Suggested Appr oachSince reading sel ections are drawn from many differentdisciplines and so urces, you should not be discouraged if you encounter m aterial with which you are not familiar. It isimportant to rem ember that questions are to be answeredexclusively on the basis of the information provided in theselection. There i s no particular knowledge that you areexpected to brin g to the test, and you should not makeinferences based on any prior knowledge of a subject thatyou may have. Yo u may, however, wish to defer working on aset of questions t hat seems particularly difcult or unfamiliaruntil after you hav e dealt with sets you nd easier.

Strategies. In preparing for the test, you shouldexperiment with different strategies and decide which workmost effectively f or you. These include:

• reading the sel ection very closely and then answeringthe questions;

• reading the qu estions first, reading the selection closely,and then return ing to the questions; or

• skimming the s election and questions very quickly,then rereading the selection closely and answeringthe questions.

Remember that your strategy must be effective for youunder timed conditions.

Reading the selection. Whatever strategy you choose,you should give the passage or pair of passages at leastone careful reading before answering the questions. Try todistinguish main ideas from supporting ideas, andopinions or attitudes from factual, objective information.

Note transitions from one idea to the next and examinethe relationships among the different ideas or parts of apassage, or between the two passages in comparativereading sets. Consider how and why an author makespoints and draws conclusions. Be sensitive to implicationsof what the passages say.

You may nd it helpful to mark key parts of passages.For example, you might underline main ideas or importantarguments, and you might circle transitional words—

“although,” “nevertheless,” “correspondingly,” and thelike—that will help you map the structure of a passage.Moreover, you might note descriptive words that will helpyou identify an author’s attitude toward a particular ideaor person.

Answering the Questions• Always read all the answer choices before selecting the

best answer. The best answer choice is th e one thatmost accurately and completely answers t he questionbeing posed.

• Respond to the specific question being a sked. Do not

pick an answer choice simply because it is a truestatement. For example, picking a true st atement mightyield an incorrect answer to a question in which you areasked to identify an author’s position on a n issue, sincehere you are not being asked to evaluate the truth of theauthor’s position but only to correctly ide ntify what thatposition is.

• Answer the questions only on the basis of the informationprovided in the selection. Your own views, interpretations,or opinions, and those you have heard fro m others, maysometimes conflict with those expressed in a readingselection; however, you are expected to w ork within thecontext provided by the reading selection. You should notexpect to agree with everything you encou nter in readingcomprehension passages.

Analytical Reasoning Questions

Analytical reasoning items are designed to m easure yourability to understand a structure of relationsh ips and to drawlogical conclusions about the structure. You are asked tomake deductions from a set of statements, r ules, orconditions that describe relationships among entities suchas persons, places, things, or events. They simulate thekinds of detailed analyses of relationships that a law studentmust perform in solving legal problems. For example, apassage might describe four diplomats sitting around atable, following certain rules of protocol as to who can sitwhere. You must answer questions about the implications of the given information, for example, who is sittingbetween diplomats X and Y.

Page 5: LSAT India 2014 Sample Question Paper 4

8/13/2019 LSAT India 2014 Sample Question Paper 4

http://slidepdf.com/reader/full/lsat-india-2014-sample-question-paper-4 5/45

3

The passage used for each group of questions describesa common relationship such as the following:

• Assignment: Two parents, P and O, and their children, Rand S, must go to the dentist on four consecutive days,designated 1, 2, 3, and 4;

• Ordering: X arrived before Y but after Z;

• Grouping: A manager is trying to form a project teamfrom seven staff members—R, S, T, U, V, W, and X. Eachstaff member has a particular strength—writing,planning, or facilitating;

• Spatial: A cer tain country contains six cities and each cityis connected to at least one other city by a system of roads, some o f which are one-way.

Careful reading and analysis are necessary to determinethe exact natur e of the relationships involved. Somerelationships are xed (e.g., P and R always sit at the sametable). Other rel ationships are variable (e.g., Q must be

assigned to eith er table 1 or table 3). Some relationshipsthat are not stat ed in the conditions are implied by and canbe deduced fro m those that are stated (e.g., if onecondition about books on a shelf species that Book L is tothe left of Book Y, and another species that Book P is tothe left of Book L, then it can be deduced that Book P is tothe left of Book Y).

No formal tra ining in logic is required to answer thesequestions corre ctly. Analytical reasoning questions areintended to be answered using knowledge, skills, andreasoning abilit y generally expected of college studentsand graduates.

Suggested Ap proachSome people m ay prefer to answer rst those questionsabout a passage that seem less difcult and then those thatseem more dif cult. In general, it is best not to start anotherpassage before nishing one begun earlier, because muchtime can be lost in returning to a passage andreestablishing fa miliarity with its relationships. Do notassume that bec ause the conditions for a set of questionslook long or co mplicated, the questions based on thoseconditions will n ecessarily be especially difcult.

Reading the passage. In reading the conditions, do notintroduce unwarranted assumptions. For instance, in a setestablishing relationships of height and weight among themembers of a team, do not assume that a person who istaller than another person must weigh more than thatperson. All the information needed to answer eachquestion is provided in the passage andthe question itself.

The conditions are designed to be as clear as possible;do not interpret them as if they were intended to trick you.

For example, if a question asks how many people could beeligible to serve on a committee, consider only thosepeople named in the passage unless directed otherwise.When in doubt, read the conditions in their most obvioussense. Remember, however, that the language in theconditions is intended to be read for precise meaning. It isessential to pay particular attention to words that describeor limit relationships, such as “only,” “exactly,” “never,”“always,” “must be,” “cannot be,” and the like.

The result of this careful reading will be a clear picture of the structure of the relationships involved, including thekinds of relationships permitted, the participants in therelationships, and the range of actions or attributesallowed by the relationships for these participants.

Questions are independent. Each que stion should beconsidered separately from the other que stions in its set;no information, except what is given in the originalconditions, should be carried over from on e question toanother. In some cases, a question will sim ply ask forconclusions to be drawn from the conditio ns as originallygiven. Some questions may, however, add information tothe original conditions or temporarily susp end one of the

original conditions for the purpose of that question only.For example, if Question 1 adds the inform ation “if P issitting at table 2 ...,” this information shou ld NOT becarried over to any other question in the g roup.

Highlighting the text; using diagrams. Many peoplend it useful to underline key points in the passage and ineach question. In addition, it may prove ve ry helpful todraw a diagram to assist you in nding the solution tothe problem.

In preparing for the test, you may wish t o experimentwith different types of diagrams. For a sch edulingproblem, a calendar-like diagram may be helpful. For aspatial relationship problem, a simple map can be auseful device.

Even though some people nd diagram s to be veryhelpful, other people seldom use them. A nd among thosewho do regularly use diagrams in solving t hese problems,there is by no means universal agreement on which kind of diagram is best for which problem or in w hich cases adiagram is most useful. Do not be concern ed if a particularproblem in the test seems to be best appr oached withoutthe use of a diagram.

Logical Reasoning Questions

Logical reasoning questions evaluate your ability tounderstand, analyze, criticize, and complete a variety of arguments. The arguments are contained in shortpassages taken from a variety of sources, including lettersto the editor, speeches, advertisements, newspaperarticles and editorials, informal discussions andconversations, as well as articles in the humanities, thesocial sciences, and the natural sciences.

Page 6: LSAT India 2014 Sample Question Paper 4

8/13/2019 LSAT India 2014 Sample Question Paper 4

http://slidepdf.com/reader/full/lsat-india-2014-sample-question-paper-4 6/45

4

Each logical reasoning question requires you to readand comprehend a short passage, then answer one ortwo questions about it. The questions test a variety of abilities involved in reasoning logically and thinkingcritically. These include:

• recognizing the point or issue of an argument ordispute;

• detecting the assumptions involved in an argumentationor chain of reasoning;

• drawing reasonable conclusions from given evidence orpremises;

• identifying and applying principles;

• identifying the method or structure of an argument orchain of reason ing;

• detecting reas oning errors and misinterpretations;

• determining ho w additional evidence or argumentationaffects an argu ment or conclusion; and

• identifying exp lanations and recognizing resolutions of conflicting fact s or arguments.

The questions do not presuppose knowledge of theterminology of fo rmal logic. For example, you will not beexpected to know the meaning of specialized terms such as“ad hominem” o r “syllogism.” On the other hand, you willbe expected to u nderstand and critique the reasoningcontained in argu ments. This requires that you possess, at aminimum, a colle ge-level understanding of widely usedconcepts such as argument, premise, assumption, andconclusion.

Suggested Appr oachRead each questi on carefully. Make sure that youunderstand the m eaning of each part of the question. Makesure that you und erstand the meaning of each answerchoice and the w ays in which it may or may not relate to thequestion posed.

Do not pick a r esponse simply because it is a truestatement. Altho ugh true, it may not answer the questionposed.

Answer each question on the basis of the information thatis given, even if you do not agree with it. Work within thecontext provided by the passage. The questions do notinvolve any tricks or hidden meanings.

TAKING THE PREPTEST UNDER SIMULATEDCONDITIONS

One important way to prepare for the LSAT—India is tosimulate the day of the test by taking a practice test underactual time constraints. Taking a practice test under timedconditions helps you to estimate the amount of time youcan afford to spend on each question in a section and todetermine the question types on which you may need

additional practice.Since the LSAT—India is a timed test, it is important touse your allotted time wisely. During the test, you may workonly on the section designated by the test supervisor. Youcannot devote extra time to a difcult section and make upthat time on a section you nd easier. In paci ng yourself,and checking your answers, you should think of each sectionof the test as a separate minitest.

Be sure that you answer every question on the test. Whenyou do not know the correct answer to a que stion, rsteliminate the responses that you know are in correct, thenmake your best guess among the remaining choices. Do notbe afraid to guess, as there is no penalty for incorrect

answers.Please note that in the LSAT—India, some sections may

consist of questions with four answer choices , while theother sections consist of questions with veanswer choices.

When you take the practice test that follow s, abide by allthe requirements specied in the directions and keepstrictly within the specied time limits. Work without a restperiod. When taken under conditions as mu ch like actualtesting conditions as possible, the practice t estprovides very useful preparation for taking th e LSAT—India.

Ofcial directions are included in this pra ctice test sothat you can approximate actual testing con ditions as youpractice. To take the test:

• Set a timer for 35 minutes. Answer all the q uestions inSECTION I. Stop working on that section w hen the 35minutes have elapsed.

• Repeat, allowing yourself 35 minutes each for sections II,III, and IV.

• An answer key is provided so that you can evaluate yourperformance on the PrepTest.

Page 7: LSAT India 2014 Sample Question Paper 4

8/13/2019 LSAT India 2014 Sample Question Paper 4

http://slidepdf.com/reader/full/lsat-india-2014-sample-question-paper-4 7/45

Please Note: The answer sheet in thi s PrepTest is not an exact r eplica of the answer sh eet used w ith t he actual test.

Page 8: LSAT India 2014 Sample Question Paper 4

8/13/2019 LSAT India 2014 Sample Question Paper 4

http://slidepdf.com/reader/full/lsat-india-2014-sample-question-paper-4 8/45

Page 9: LSAT India 2014 Sample Question Paper 4

8/13/2019 LSAT India 2014 Sample Question Paper 4

http://slidepdf.com/reader/full/lsat-india-2014-sample-question-paper-4 9/45

• Analytical Reasoning.......................................SECTION I

• Logical Reasoning ..................................... ......SECTION II

• Reading Comprehension................................SECTION III

• Logical Reasoning ..................................... ......SECTION IV

THE PREPTEST

7

Page 10: LSAT India 2014 Sample Question Paper 4

8/13/2019 LSAT India 2014 Sample Question Paper 4

http://slidepdf.com/reader/full/lsat-india-2014-sample-question-paper-4 10/45

SECTION I

Time—35 minutes

20 Questions

Directions: Each group of questions in this section is based on a set of conditions. In answering some of the questions, it may beuseful to draw a rough diagram. Choose the response that most accurately and completely answers each question and blacken

the corresponding space on your answer sheet.

Questions 1–6

Meena has six colored pencils—green, maroon, orange, red,white, and yello w. She sketches a drawing using exactly fiveof the six pencil s, one pencil at a time. The followingconditions must hold:

No pencil is used more than once.Either the m aroon or the yellow pencil is not used.The red penc il is used at some time before the green.The red penc il is used at some time before either theorange or th e yellow, but not before both.

The green pe ncil is used at some time before either themaroon or t he white, but not before both.

1. Which one of the following could be an accurate list of the pencils used, from first to fifth?

(A) orange, green, red, maroon, yell ow(B) red, green, orange, white, maroo n(C) red, orange, maroon, white, gree n(D) white, red, orange, green, maroo n(E) white, yellow, orange, green, red

GO ON TO TH E NEXT PAGE.

-8-1 11

Page 11: LSAT India 2014 Sample Question Paper 4

8/13/2019 LSAT India 2014 Sample Question Paper 4

http://slidepdf.com/reader/full/lsat-india-2014-sample-question-paper-4 11/45

2. Which one of the following could be true?

(A) The maroon pencil is used first, whereas thewhite is used second.

(B) The white pencil is used first, whereas the greenis used second.

(C) The yellow pencil is used first, whereas the

orange is used second.(D) The yellow pencil is used first, whereas the

orange is used fourth.(E) The yellow pencil is used fourth, whereas the

maroon is used fifth.

3. Which one of the following CANNOT be the color of the fifth pe ncil used?

(A) gre en(B) ma roon(C) ora nge(D) wh ite(E) yel low

4. Which one of the following could be an accurate list of the pencils used, from first to fifth?

(A) ma roon, green, white, red, orange(B) ma roon, red, green, white, yellow(C) red , green, orange, white, yellow(D) red , white, orange, green, maroon(E) wh ite, red, maroon, green, orange

5. Which one of the following CANNOT be the color of the third pencil used?

(A) green(B) maroon(C) red(D) white

(E) yellow

6. Suppose that the condition that either the maroon or theyellow pencil is not used no longer holds. If all other conditions remain in effect, then which one of thefollowing could be an accurate list of th e pencils used,from first to fifth?

(A) maroon, red, green, orange, yell ow(B) maroon, yellow, red, green, whit e(C) orange, green, red, yellow, maro on(D) orange, red, maroon, green, whit e(E) white, red, green, maroon, yello w

GO ON TO TH E NEXT PAGE.

-9- 11 1

Page 12: LSAT India 2014 Sample Question Paper 4

8/13/2019 LSAT India 2014 Sample Question Paper 4

http://slidepdf.com/reader/full/lsat-india-2014-sample-question-paper-4 12/45

Questions 7–12

A bird sanctuary contains birds of at least two of thefollowing six kinds—kingfishers, mynas, peacocks, robins,sparrows, and woodpeckers. No other kinds of birds are in thesanctuary. The following conditions must hold:

Woodpeckers are not in the sanctuary if mynas are.Peacocks are not in the sanctuary if robins are.Either robins or woodpeckers, but not both, are in thesanctuary.

Woodpeckers are in the sanctuary if kingfishers, sparrows, or both are in the sanctuary.

7. Which one of the following could be a complete andaccurate list of the kinds of birds in the sanctuary?

(A) kingfishers, mynas(B) peacocks, woodpeckers(C) kingfishers, robins, sparrows(D) mynas, peacocks, robins

(E) mynas, peacocks, sparrows

GO ON TO TH E NEXT PAGE.

-10-1 11

Page 13: LSAT India 2014 Sample Question Paper 4

8/13/2019 LSAT India 2014 Sample Question Paper 4

http://slidepdf.com/reader/full/lsat-india-2014-sample-question-paper-4 13/45

8. What is the maximum number of the kinds of birdsthat could be in the sanctuary?

(A) two(B) three(C) four (D) five

(E) six

9. If both kingfishers and sparrows are in the sanctuary,then each of the following could be true EXCEPT:

(A) Exa ctly three kinds of birds are in the sanctuary.(B) Exa ctly four kinds of birds are in the sanctuary.(C) The re are peacocks in the sanctuary.(D) The re are robins in the sanctuary.(E) The re are woodpeckers in the sanctuary.

10. If there are no kingfishers in the sanctuary, then whichone of the following must be true?

(A) The re are no sparrows in the sanctuary.(B) The re are no woodpeckers in the sanctuary.(C) The re are peacocks in the sanctuary.(D) At least three kinds of birds are in the sanctuary.(E) At most three kinds of birds are in the sanctuary.

11. If there are no robins in the sanctuary, then which one of the following is a kind of bird that CANNOT be in thesanctuary?

(A) kingfishers(B) mynas(C) peacocks

(D) sparrows(E) woodpeckers

12. If peacocks are in the sanctuary, then which one of the following must be true?

(A) Exactly two kinds of birds are in the sanctuary.(B) Exactly three kinds of birds are i n the sanctuary.(C) Exactly four kinds of birds are i n the sanctuary.(D) There are no kingfishers in the s anctuary.(E) There are no mynas in the sanct uary.

GO ON TO TH E NEXT PAGE.

-11- 11 1

Page 14: LSAT India 2014 Sample Question Paper 4

8/13/2019 LSAT India 2014 Sample Question Paper 4

http://slidepdf.com/reader/full/lsat-india-2014-sample-question-paper-4 14/45

Questions 13–16

Last year, seven stores—Faroj’s Appliances, Good Buy,Intelligent Office, LANs for Less, Networks Unlimited,Silicon Village, and Uptown Computing—each sold exactly100, 200, 300, 400, or 500 computers, consistent with thefollowing conditions:

Good Buy sold exactly the same number of computers as Networks Unlimited did.Intelligent Office sold exactly the same number of computers as Uptown Computing did.

Neither Faro j’s Appliances nor Silicon Village sold exactlythe same nu mber of computers as any other store.

LANs for Le ss sold more computers than UptownComputing did.

Intelligent O ffice sold more computers than Good Buydid.

13. Which one of the following could be an accuratematching of the stores to the exact number of computerseach sold last year?

(A) Faroj’s Appliances: 100Good Buy: 200Intelligent Office: 300

LANs for Less: 400 Networks Unlimited: 200Silicon Village: 500Uptown Computing: 300

(B) Faroj’s Appliances: 100Good Buy: 200Intelligent Office: 400LANs for Less: 300 Networks Unlimited: 200Silicon Village: 500Uptown Computing: 400

(C) Faroj’s Appliances: 500Good Buy: 200Intelligent Office: 300LANs for Less: 400 Networks Unlimited: 500Silicon Village: 100Uptown Computing: 300

(D) Faroj’s Appliances: 500Good Buy: 300Intelligent Office: 200LANs for Less: 400 Networks Unlimited: 300Silicon Village: 100Uptown Computing: 200

(E) Faroj’s Appliances: 500Good Buy: 300

Intelligent Office: 400LANs for Less: 200 Networks Unlimited: 300Silicon Village: 500Uptown Computing: 100

GO ON TO TH E NEXT PAGE.

-12-1 11

Page 15: LSAT India 2014 Sample Question Paper 4

8/13/2019 LSAT India 2014 Sample Question Paper 4

http://slidepdf.com/reader/full/lsat-india-2014-sample-question-paper-4 15/45

14. Which one of the following stores CANNOT have soldexactly 400 computers last year?

(A) Faroj’s Appliances(B) Good Buy(C) Intelligent Office(D) Silicon Village

(E) Uptown Computing

15. Which one of the following stores CANNOT have soldexactly 200 computers last year?

(A) Far oj’s Appliances(B) Go od Buy(C) Inte lligent Office(D) LA Ns for Less(E) Sili con Village

16. Which one of the following must be true?

(A) Faroj’s Appliances sold more computers last year than Good Buy did.

(B) Intelligent Office sold more computers last year than Silicon Village did.

(C) LANs for Less sold more computers last year

than Faroj’s Appliances did.(D) Silicon Village sold more computers last year

than Good Buy did.(E) Uptown Computing sold more computers last

year than Networks Unlimited did.

GO ON TO TH E NEXT PAGE.

-13- 11 1

Page 16: LSAT India 2014 Sample Question Paper 4

8/13/2019 LSAT India 2014 Sample Question Paper 4

http://slidepdf.com/reader/full/lsat-india-2014-sample-question-paper-4 16/45

Questions 17–20

Arbutus College owns exactly four houses that it leases tofaculty or students. Of these houses, no two are exactly thesame distance from Arbutus’s campus, and each house iseither a student house (occupied entirely by students) or afaculty house (occupied entirely by faculty). The lease lengthfor each house is one, two, or three semesters. The followingconditions must hold:

No student house has a three-semester lease.At least two houses each have longer leases than doesthe house c losest to campus.

Every stude nt house (if there are any) is farther fromcampus tha n any faculty house (if there are any).

17. Which one of the following could be a complete andaccurate list of the leases of the student houses, orderedfrom the student house closest to campus to the studenthouse farthest from campus?

(A) one semester, one semester (B) two semesters, three semesters

(C) one semester, two semesters, one semester (D) one semester, two semesters, two semesters, one

semester (E) one semester, two semesters, three semesters,

two semesters

GO ON TO TH E NEXT PAGE.

-14-1 11

Page 17: LSAT India 2014 Sample Question Paper 4

8/13/2019 LSAT India 2014 Sample Question Paper 4

http://slidepdf.com/reader/full/lsat-india-2014-sample-question-paper-4 17/45

18. If the house farthest from campus has a lease longer than that of each of the other houses, then which oneof the following could be true?

(A) Each faculty house has a two-semester lease.(B) Exactly two houses each have a one-semester

lease.

(C) Exactly three houses each have a two-semester lease.

(D) None of the houses has a one-semester lease.(E) None of the houses has a two-semester lease.

19. What is the maximum number of houses that could all bestudent hou ses with two-semester leases?

(A) zer o(B) one(C) two(D) thre e(E) fou r

20. Which one of the following must be true of the twohouses that are neither the house farthest from campusnor the house closest to campus?

(A) At least one of them has a lease the same lengthas that of the house closest to campus.

(B) At least one of them has a lease longer than that

of the house closest to campus.(C) At least one of them has a lease shorter than three

semesters.(D) Neither of them is a student house.(E) Both of them are student houses.

S T O PIF YOU FIN ISH BEFORE TIME IS CALLED, YOU MAY CHECK YOUR WORK ON THIS SECT ION ONLY.

DO NOT WORK ON ANY OTHER SECTION IN THE TEST.

-15- 11 1

Page 18: LSAT India 2014 Sample Question Paper 4

8/13/2019 LSAT India 2014 Sample Question Paper 4

http://slidepdf.com/reader/full/lsat-india-2014-sample-question-paper-4 18/45

SECTION II

Time—35 minutes

23 Questions

Directions: The questions in this section are based on the reasoning contained in brief statements or passages. For somequestions, more than one of the choices could conceivably answer the question. However, you are to choose the best answer; that

is, the response that most accurately and completely answers the question. You should not make assumptions that are bycommonsense standards implausible, superfluous, or incompatible with the passage. After you have chosen the best answer, blacken the corresponding space on your answer sheet.

1. Jay: The development of a plain writing style inseve nteenth-century England was mainly due toan increase in the literacy rate. To reach moderatelyeduc ated readers, writers simplified their style.

Chandra: N o, the pivotal factor was the increasinginter est in science among the most highlyeduc ated people; a complex, artificial style,how ever beautiful, interfered with the pres entation of scientific facts.

Jay’s and C handra’s comments indicate that theydisagree ab out

(A) wh ether the quality of written works inseventeenth-century England increased or decreased as a result of the development of a plain writing style

(B) the extent of the changes in writing style thattook place in seventeenth-century England

(C) wh ether there was an increase in the percentageof people who were able to read in Englandduring the seventeenth century

(D) ho w widespread the dissemination of scientificknowledge in seventeenth-century England was

(E) wh at was the primary cause of the developmentof a plain writing style in seventeenth-centuryEngland

2. On the nights immediately following the mysteriousTunguska event, which destroyed a tract of Siberianwilderness in 1908, eyewitnesses reporte d seeingnoctilucent clouds—brilliant night-visible clouds madeup of ice particles that form rarely and o nly at veryhigh altitudes. Recently, noctilucent clou ds have beenobserved on the nights following launch es of rocketsthat release large amounts of water vapo r into the upper atmosphere. This shows that it was a co met impact andnot the impact of an asteroid that caused the destructionin Siberia.

The argument depends on assuming whi ch one of thefollowing?

(A) Comets but not asteroids release large amountsof water vapor into the upper a tmospherewhen they collide with Earth.

(B) Noctilucent clouds are visible fo r manyconsecutive nights following th e release of water vapor into the upper atm osphere.

(C) Comets collide with Earth more frequentlythan asteroids do.

(D) Eyewitnesses have reported seei ng noctilucent

clouds after asteroids have colli ded with Earth.(E) The fact that noctilucent clouds are made of ice particles in the upper atmosphe re was onlyrecently discovered.

GO ON TO TH E NEXT PAGE.

-16- 2 2 22

Page 19: LSAT India 2014 Sample Question Paper 4

8/13/2019 LSAT India 2014 Sample Question Paper 4

http://slidepdf.com/reader/full/lsat-india-2014-sample-question-paper-4 19/45

3. Children in the first six standards of school who are publicly singled out for enthusiastic praise by their teachers show marked improvement in their schoolwork.But students in higher standards tend to react negativelyto such praise, and their academic performance tends todecline as a result.

Which one of the following most helps to explain thediffering reactions described above?

(A) Younger children respond more to the tone of voice with which criticism is offered than toits explicit content.

(B) Old er students are less concerned with theap proval of teachers than with the details of the criticisms and suggestions the teachers make.

(C) Old er students are more likely than younger students to believe that there are better indicatorsof their academic performance than the gradesthey earn in class.

(D) Old er students believe that their teachers’ praise

will inspire the disapproval of their peers, towhich they have a greater aversion than doyounger students.

(E) Yo unger students are more concerned with pu blic appearance than are older students.

4. Bunty: Th e primary job of police officers is keepingthe peace. Since their subsidiary jobs, such ascont rolling traffic, hinder their performance of the prim ary one, people other than police officersshou ld be hired to perform the subsidiary jobs.

Naina: To perform their primary job well, policeoffic ers must have the trust of citizens. They gain

that trust by performing their subsidiary jobs,whic h are the only functions most citizens seethem fulfill.

Bunty and Naina disagree with each other about

(A) wh ether the primary job of police officers iskeeping the peace

(B) wh ether the subsidiary jobs police officers perform enable them to perform their primary jo b effectively

(C) wh ether police officers need to win the trustof law-abiding citizens in order to keep the peace effectively

(D) whether police officers are performing their

primary jobs as well as they should be(E) whether police officers can effectively gain the

trust of citizens

5. Any organization must consider changing its basicstructure if there is a dramatic change in its size.Doubling or tripling in size can lead to inefficiency andmismanagement, which restructuring often alleviates.Experience further shows that the more dramatic thechange in size, the more fundamental the restructuringneeds to be. Therefore, any organization must consider changing its basic structure once it is 20 years old.

The argument’s conclusion follows logically if whichone of the following is assumed?

(A) No organization that has remain ed the same sizefor 20 years is efficient.

(B) Any organization that has been r estructured is prepared to double or triple in size.

(C) No organization that has not cha nged in sizeneeds restructuring.

(D) Any organization that has existe d for 20 yearshas undergone a dramatic chan ge in size.

(E) No organization that has not bee n restructured

is as efficient as any organizati on that has been restructured.

GO ON TO TH E NEXT PAGE.

-17-2 2 22

Page 20: LSAT India 2014 Sample Question Paper 4

8/13/2019 LSAT India 2014 Sample Question Paper 4

http://slidepdf.com/reader/full/lsat-india-2014-sample-question-paper-4 20/45

6. If deep-sea divers ascend too rapidly from ocean depths,the decreasing pressure causes nitrogen to be released asgas bubbles in the blood. This condition is known as“the bends.” Sea snakes, who, like humans, breathe air that contributes nitrogen to the blood, are neverthelessable to make extremely rapid ascents and descents inocean waters without suffering from the bends.

Which one of the following, if true, contributes most toan explanation of the difference described betweenhumans and sea snakes?

(A) Sea snakes, unlike humans, can excrete nitrogenfr om their blood into the sea by means of extremely small blood vessels beneath their skin.

(B) Hu man deep-sea divers are trained to makeseveral stops on ascending from deep water inor der to adjust to decreasing pressure gradually,whereas sea snakes ascend from ocean depthswithout stopping.

(C) The lung of the sea snake extends from its

head to its tail and, when inflated, occupiesabout 10 percent of the volume of the seasnake’s body.

(D) A r apid release of bubbles of nitrogen gas intothe blood inhibits circulation and can depriveor gans of the blood they need to function.

(E) The rapidity with which sea snakes are able todescend or ascend in ocean water is partiallydetermined by the degree of buoyancy permitted by the inflation of the sea snake’s lung.

7. Knowledge in all fields is expanding and Ph.D. studentstake longer than ever before—sometimes eight years— to complete degree requirements. Yet, instead of agreeingthat the lon ger period is needed, some noted scholars arerecommend ing that Ph.D. programs reduce their requirement s and have students finish within three years.

Which one of the following, if true, would mostcontribute t o a justification of the noted scholars’recommend ation?

(A) Qu ality of research matters more than quantity,even though, on average, the more time aPh.D. student spends on research, the greater the quantity of research produced.

(B) So me unusually talented Ph.D. students alreadycomplete all Ph.D. requirements within three

years.(C) For at least the last 50 years, no researcher has been able to be conversant with any more thana small fraction of the existing knowledgewithin any given field.

(D) Many outstanding scholars in the past haveachieved great things in their fields withoutever having a Ph.D. or equivalent degree.

(E) The most important objectives of Ph.D. programs can be adequately fulfilled with thereduced requirements recommended.

8. Loggerhead turtles are an endangered species. Aquariumofficials presumably know and are concerned about thedeclining number of wild loggerheads. Nevertheless,aquariums keep loggerheads and display them. Theseturtles are being kept in captivity and are thus preventedfrom adding to the population of wild turtles.

Each of the following, if true, contributes to an explanationof why aquarium officials keep loggerheads in captivityEXCEPT:

(A) The adult loggerheads in captivity are tooseverely injured to survive in th e wild.

(B) The baby loggerheads in captivit y are hatchlingstoo weak to survive in the wild .

(C) The population of loggerheads i n captivity hasdeclined slightly over the last 2 0 years.

(D) The captive loggerheads are use d to helpeducate the public about the ne eds of wildloggerheads so that the public will take greater care not to harm them.

(E) The captive loggerheads are bre d, and their offspring are released into the wild, whichhelps increase the number of w ild turtles.

GO ON TO TH E NEXT PAGE.

-18- 2 2 22

Page 21: LSAT India 2014 Sample Question Paper 4

8/13/2019 LSAT India 2014 Sample Question Paper 4

http://slidepdf.com/reader/full/lsat-india-2014-sample-question-paper-4 21/45

9. Politician: Members of the national legislature havereceived a very large number of phone calls andletters from people wanting to express their opinions about the new bill before the legislature,which would increase the current tax on bidis andcigarettes by one rupee per pack to pay for newantismoking advertisements. Since a greatmajority of these people expressed opposition tothe bill, it is clear that the public overwhelminglyopposes this tax increase.

Which one of the following, if true, most seriouslyweakens th e argument?

(A) Peo ple who do not smoke bidis or cigarettes but work in tobacco-related industries are justas likely as smokers to oppose a bidi andcigarette tax.

(B) Inc reasing the tax on bidis and cigarettes by oneru pee per pack would reduce bidi and cigaretteconsumption so that total revenue from such

taxes would not increase.(C) Peo ple who feel strongly about a particular bill

ar e more likely to express their opinions to alegislator by phone than by letter.

(D) Mo st antismoking advertisements are currently paid for by private organizations rather than being funded by taxes paid to the government.

(E) Peo ple who oppose a tax bill are far more likelyto contact a legislator to express their opinionsthan are those who do not oppose the bill.

10. Anju: The Adkjos corporation does not fund social prog rams. Therefore, although Adkjos does makefine products, it is not socially responsible.

Sanjeev: T hat doesn’t mean that Adkjos is not sociallyresp onsible. If a business offers good pay and bene fits to its employees, and fine service andvalu e to customers, it is socially responsible.Adk jos does those things.

On the bas is of their statements, Anju and Sanjeevare commit ted to disagreeing about which one of thefollowing?

(A) To be socially responsible, it is not enough for a company to make fine products.

(B) Soc ially responsible companies offer better paythan companies that are not socially responsible.

(C) Not all companies that make fine products fundsocial programs.

(D) Funding social programs is required for a business to be socially responsible.

(E) Adkjos treats its employees and customers well.

11. Two crucial claims of relativity theory can be directlyconfirmed. Utilizing elementary particles in high-energyaccelerators, we can demonstrate that at high velocitiesobjects are subject to time dilation and an increase inmass. However, a third claim of the theory, the Lorentzcontraction thesis, which is connected to the other two,is not directly confirmable. But the fact that the theoryin general is supported by experimental results indirectlyconfirms the contraction thesis.

Which one of the following most accurately expressesthe principle underlying the reasoning above?

(A) A thesis that is related to other t heses can beindirectly confirmed by the dire ct verificationof the others.

(B) The theses of a physical theory t hat cannot beconfirmed by observable pheno mena can only be confirmed indirectly.

(C) One must confirm all the theses of a theory before accepting the theory gen erally.

(D) Any theory that is not inconsiste nt withexperimental results is acceptab le.

(E) A thesis that is not directly conf irmable should be counted as confirmed only w hen thetheory of which it is part is sup ported byexperimental results.

GO ON TO TH E NEXT PAGE.

-19-2 2 22

Page 22: LSAT India 2014 Sample Question Paper 4

8/13/2019 LSAT India 2014 Sample Question Paper 4

http://slidepdf.com/reader/full/lsat-india-2014-sample-question-paper-4 22/45

12. Exposure to a large dose of something that causes bodily damage—such as excessive heat, poison, or nuclear radiation—is of course harmful to an organism.But, surprisingly, exposure to small doses of suchstressors has been shown to extend life span in variousspecies, including fruit flies, protozoans, worms,and rodents.

Which one of the following, if true, most helps toexplain the surprising phenomenon described above?

(A) In most of the species in which exposure tosmall doses of stressors increases longevity,the increase is so small that it is barelymeasurable.

(B) Ex posure to small doses of stressors stimulatesan organism’s natural repair mechanisms to fixany damage caused by the stressors as well assome unrelated damage.

(C) Ex posure to a given dose of a poison or other str essor may cause more serious damage to

some members of a species than to others.(D) Re peated exposure to a stressor is much more

lik ely than a single exposure to cause permanent damage to an organism.

(E) Eve n a substance that is extremely toxic willnot harm an organism if the organism isex posed to only an extremely small quantityof the substance.

13. Most peopl e prefer to hire people they know. Thus it isdifficult for people without business contacts to findgood jobs. The only way to help such people find jobsis through nonfinancial incentives to change employers’ behavior.

Which one of the following most accurately describesthe role pla yed in the argument by the claim that most people pref er to hire people they know?

(A) It i s cited as an explanation of why employersnever hire those whom they do not know.

(B) It i s cited as an example of the general principlethat nonfinancial incentives to change employers’ behavior are necessary.

(C) It i s a conclusion for which the only supportof fered is that there is a need for nonfinancialincentives to change employers’ behavior.

(D) It is a hypothesis to which the claim that people

without business contacts have difficulty infinding good jobs is offered as an objection.(E) It is a premise offered in support of the claim

that people without business contacts havedifficulty in finding good jobs.

14. An energy company proposes placing 250 giantturbines into the Gulf Stream to generate electricity for North America. Some Europeans worry, however,about the potential threat to their climate. Without thewarming effects of the Gulf Stream, much of Europewould be as cold as Labrador and the Yukon, areas atthe same latitude that are unaffected by warmingcurrents. However, their concern is unwarranted: thecompany claims that 250 turbines would slow theGulf Stream about 1 percent, which is not enough toaffect the European climate.

Which one of the following most accura tely describes aflaw in the reasoning of the argument?

(A) The argument relies on an autho rity that may be biased.

(B) The argument presumes, without providing justification, that latitude and te mperatureare linked.

(C) The argument takes for granted that Europe’s

climate is more important than meeting theenergy needs of North America .

(D) The argument ignores the potent ial threat tomarine life posed by placing tu rbines inthe ocean.

(E) The conclusion of the argument contradicts atleast one of its premises.

GO ON TO TH E NEXT PAGE.

-20- 2 2 22

Page 23: LSAT India 2014 Sample Question Paper 4

8/13/2019 LSAT India 2014 Sample Question Paper 4

http://slidepdf.com/reader/full/lsat-india-2014-sample-question-paper-4 23/45

15. Like airplane accidents, most accidents in medical careresult from human error, particularly failures incommunication, leadership, and decision making.Following the introduction of standard proceduresgoverning these three areas, the airline industrysucceeded in significantly reducing the number andseverity of accidents. Since airplane flights and healthcare delivery both require the cooperation of groups of people with diverse knowledge and skills, the medicalcare community should adopt a similar set of proceduresin order to reduce errors.

Which one of the following is a technique of reasoningused in the argument?

(A) def ending a general principle by presenting twose parate successful applications of that principle

(B) just ifying the introduction of a set of procedures by outlining undesirable results in ananalogous situation in which those procedureswere not followed

(C) atte mpting to undermine a generalization by pr oviding a counterexample to that generalization

(D) arg uing for taking a course of action basedon results of taking such action in ananalogous situation

(E) pro viding examples in order to show thattwo seemingly dissimilar situations are infact the same

16. In a certain democratic country the legislature passed anew tax la w over the principled objections of the parliamenta ry opposition. Some opposition leaders brokethe new la w by refusing to pay the new tax. Theydefended th eir lawbreaking by citing the historical precedent i n the country of activists’ having to break laws in win ning for women the right to vote.

Which one of the following, if true, most weakens theopposition leaders’ argument in defense of their actions?

(A) Alt hough they had principled objections to thenew law, the opposition leaders derived a personal monetary benefit from breaking the law.

(B) The activists fought for equality of the sexes,a principle easier to define than the goal pursued by the opposition leaders.

(C) The opposition leaders, unlike the activists,were part of the democratic process that they

are defying.(D) The opposition leaders, unlike the activists, broke the law in a way that did not directlylead to public confrontations with lawenforcement personnel.

(E) The opposition leaders, unlike the activists,fought for a return to an earlier state of affairs.

17. Small species, such as fathead minnows and waterfleas,can be used to uncover the quantity of toxic chemicals being dumped into rivers and streams. Under new rulesissued in a certain country, these “indicator” species will be used to test the effluent pouring out of sewagetreatment plants and factories among the country’swaterways. If many of these animals die as a result of the test, the country is going to require that pollutionlevels be reduced. The new rules will apply to about450 of the 2,000 factories and sewage treatment plantsthat legally discharge into the country’s rivers and streams.

Which one of the following can be infer red from the passage above?

(A) Under the new rules, certain sm all species will be introduced into water sampl es taken from2,000 sites around the country.

(B) If, after the test, the country doe s not requirethat pollution levels be reduced , then few or none of the individuals of the i ndicator species

died as a result of the test.(C) If few individuals of the indicato r species die

during the test period, the coun try will not takeaction on water pollution.

(D) In the country in question, there are 1,550factories and sewage treatment plants thatdischarge pollutants illegally.

(E) Under the new rules, 450 factori es and sewagetreatment plants will not be per mitted todischarge into the country’s riv ers and streams.

GO ON TO TH E NEXT PAGE.

-21-2 2 22

Page 24: LSAT India 2014 Sample Question Paper 4

8/13/2019 LSAT India 2014 Sample Question Paper 4

http://slidepdf.com/reader/full/lsat-india-2014-sample-question-paper-4 24/45

18. Aesthetician: Zahib’s rejection of contemporaryliterature’s aesthetic value depends on his claimthat today’s writing generally fails to grappleseriously enough with life’s deepest ethicalquestions—whereas great books, he maintains, present profound moral lessons and “the stuff of conscience.” But what resounding moral lessondoes Vikram Seth’s A Suitable Boy or Devaki Nandan Khatri’s Chandrakanta impart? Peopleread these two great novels because they areengaging, even thrilling, stories. The absence of a pr ofound moral lesson in no way detracts fromthe aesthetic value of a novel.

The aesthet ician criticizes Zahib’s position by arguingthat it depe nds on the questionable premise that

(A) a novel that presents a profound moral lessoncan have aesthetic value

(B) tod ay’s writing generally fails to confront deepethical questions

(C) for a literary work to have aesthetic value itmust present a profound ethical message

(D) onl y novels that have aesthetic value fail to pr esent profound moral lessons

(E) the re is no distinction between engaging storiesand profound moral lessons

19. Paternalism is the practice by some governments of imposing r egulations meant to promote their citizens’own good r egardless of whether the citizens consent tothose regul ations. However, many members of freesocieties di sapprove when individual freedom iscurtailed fo r the sake of what the government deems to be the citiz ens’ own good. They generally believe thatthey, not th e government, know what activities are intheir best i nterest.

If the state ments above are true, which one of thefollowing must also be true?

(A) The good of citizens is usually not advanced by the practice of paternalism.

(B) The goals of free societies and the goals of their citizens always conflict.

(C) No truly free societies have governments that pr actice paternalism.

(D) In f ree societies, many of a government’scitizens disapprove of their government’s acts

of paternalism.(E) In free societies, many of a government’scitizens know what activities are in their own best interests better than their government does.

20. Historian: Concern with achievement and power surgeddramatically in the latter part of the eighteenthcentury, which is around the time that theIndustrial Revolution began in Europe. So, it isclear that this surge in concern with achievementand power was a result of the Industrial Revolution.

The reasoning in the argument is flawed because(A) it fails to adequately take into account that the

beginning of the Industrial Revolution in Europecannot be located with any great precision

(B) it fails to consider that there was some concernwith achievement and power be fore theIndustrial Revolution

(C) increasing concern with achieve ment and power may very well have been a cau se of theIndustrial Revolution

(D) there may very well have been s urges inconcern with achievement and power at timesother than during the Industrial Revolution

(E) it ignores the fact that the Indust rial Revolutiondid not reach its full maturity u ntil thenineteenth century

21. Many famous painters employ prelimina ry sketches before embarking on the final version of their work.Yet frequently these preliminary sketches are beautifuland accomplished works of art in their o wn right.Museums with small budgets will displa y these preliminary works instead of what the ar tists consider to be their finished works of art.

Which one of the following propositions is bestillustrated by the situation described abo ve?

(A) Artists are not the best judges of the value of their own work.

(B) Museums with small budgets ca nnot be asinteresting as those with large b udgets.

(C) A finished work of art cannot be producedwithout the execution of a high -quality preliminary sketch.

(D) Artifacts may have uses differen t from thoseintended by their creators.

(E) Artists’ preliminary sketches are as beautiful asthe final versions of their work .

GO ON TO THE NEXT PAGE.

-22- 2 2 22

Page 25: LSAT India 2014 Sample Question Paper 4

8/13/2019 LSAT India 2014 Sample Question Paper 4

http://slidepdf.com/reader/full/lsat-india-2014-sample-question-paper-4 25/45

22. In a recent experiment, half the subjects were expertchess players; the other half had never played. First,the subjects were shown chessboards at various stagesof actual games for ten seconds. Each expert was able torecall the positions of the pieces at least twice as well asany nonplayer. Second, the subjects were shownchessboards with the pieces randomly arranged, againfor ten seconds. In this case the experts recalled the positions only marginally better, on average, than didthe nonplayers.

Which one of the following is most strongly supported by the resu lts of the experiment?

(A) Peo ple who play chess have better memoriesthan people who do not play chess.

(B) Peo ple who play chess do not have better memories than people who do not play chess.

(C) Peo ple remember information better when theycan organize it according to rules that they know.

(D) An expert is more likely than a nonexpert to

attempt to assimilate new informationaccording to known rules and principles.

(E) Ten seconds is not long enough to memorizerandom information.

23. Professor: Different countries have different economicsystems, but all economic systems have prosperityas their primary goal. Because people all want thesame thing, there cannot be radical disagreementamong people from different economic systemsabout practical economic matters. Thus allapparent disagreement in practical economicissues is illusory.

Which one of the following most accurately describesa reasoning error in the professor’s argument?

(A) The argument contradicts itself a bout whether there are in fact differences bet ween economicsystems in different countries.

(B) The argument bases a general co nclusion abouteconomic systems on one exam ple of such asystem, which there is reason to think is atypical.

(C) The argument depends on using the key term“economic system” in two inco mpatible senses.

(D) The argument fails to justify its presumption

that the source of all apparent d isagreement in practical economic issues can b e discovered.

(E) The argument ignores the possib ility that groupsof people may share the same g oal but notagree about how best to achiev e that goal.

S T O PIF YOU FIN ISH BEFORE TIME IS CALLED, YOU MAY CHECK YOUR WORK ON THIS SECT ION ONLY.

DO NOT WORK ON ANY OTHER SECTION IN THE TEST.

-23-2 2 22

Page 26: LSAT India 2014 Sample Question Paper 4

8/13/2019 LSAT India 2014 Sample Question Paper 4

http://slidepdf.com/reader/full/lsat-india-2014-sample-question-paper-4 26/45

SECTION III

Time—35 minutes

24 Questions

Directions: Each set of questions in this section is based on a single passage or a pair of passages. The questions are to beanswered on the basis of what is stated or implied in the passage or pair of passages. For some of the questions, more than one

of the choices could conceivably answer the question. However, you are to choose the best answer; that is, the response thatmost accurately and completely answers the question, and blacken the corresponding space on your answer sheet.

The effects of the introduction of language uponhuman behavior are easily surmised. Languageallowed r ules for proper behavior to be communicatedto childre n by precept and enforced among adults by

(5) gossip. E ffective responses to different circumstanceswere usu ally guaranteed by traditional rules of behavior, and frictions were minimized becauseeveryone knew what to expect of others in allordinary situations. Such knowledge minimized

(10) quarrels, maximized effective cooperation, andallowed a n increasingly complex division of labor among la rge numbers of individuals who spoke thesame lan guage. Language, however, also makes iteasier to improve our behavioral responses when

(15) experienc e disappoints expectation. It helps us tomove to a nd fro in imagined time, rememberinguseful thi ngs from the past and planning what to do inthe future .

Lang uage is so powerful and pervasive in human(20) lives that it seems right to say that our ancestors

became f ully human only when they began to uselanguage and to act not on the basis of sensory stimulialone but in accordance with plans and hopes andverbally f ormulated ideas about themselves and

(25) everythin g around them. Once the face-to-face verbalcoordinat ion of most everyday behavior had beenachieved, major subsequent landmarks of humanhistory d epended principally on improvements incommuni cation that allowed messages to travel

(30) farther an d more accurately across time and distancethan spok en words ordinarily do.

Netw orks of messages, delivered in verbal form,suppleme nted by gesture, and sometimes solemnized by ritual, created and sustained local human

(35) communi ties. Eventually an increasing number of these pri mary communities came to be incorporatedinto large r communication networks centered in citiesand sustaining what we call civilizations. Ascompared to primary communities, civilizations were

(40) tumultuous and unstable social structures, but theywere also more powerful, exerting control over thenatural environment and coordinating the actions of larger numbers of persons partly by obedience tocommands, and partly by negotiated, more or less

(45) voluntary, exchanges of goods and services. Ever since the first civilizations arose, civilized socialcomplexity has tended to spread, until in our owntime almost all humankind is caught up in a singleglobal system, furiously exchanging messages and

(50) upsetting traditional ways of life almost everywhere.

The details of how small roving bands with onlysporadic outside contacts evolved into today’s unifiedworld cannot be fully known. Howeve r, animaginative historian can hope to disce rn major

(55) landmarks in the civilizing process by focusing on breakthroughs in communication that a ltered therange and flow of messages among hu man populations, thereby accelerating the p ropagation of novelties and meeting human needs an d wants better

(60) than before.

1. Which one of the following most accura tely expressesthe main point of the passage?

(A) The pace of modern communica tion has upsettraditional ways of life through out the world.

(B) Our ancestors became fully hum an only whenthey began to use language to move to and froin imagined time.

(C) The power of human communic ation willinevitably overcome traditional ways of lifeand create a single global syste m.

(D) The details of the history of lang uage cannot befully known, but the behavioral effects of theintroduction of language are ea sily surmised.

(E) The development of human civil ization wasfundamentally dependent on lan guage andcommunication.

2. According to the passage, one way in w hich urbancivilizations coordinate the behavior of l arge numbers of people more powerfully than primary co mmunities canis through

(A) gossip(B) ritual(C) police power (D) negotiated transactions(E) division of labor

GO ON TO THE NEXT PAGE.

33 -24- 333

Page 27: LSAT India 2014 Sample Question Paper 4

8/13/2019 LSAT India 2014 Sample Question Paper 4

http://slidepdf.com/reader/full/lsat-india-2014-sample-question-paper-4 27/45

3. By “imagined time” (line 16) the author most likelymeans

(A) contrary-to-fact situations(B) time prior to historical records(C) the time of legends and myths(D) mental representations of the past and future

(E) occasions when experience disappoints expectation

4. All of the following meet the author’s standard for identifying major landmarks of human history EXCEPT:

(A) the development of ornamental calligraphy(B) the development of the alphabet(C) the invention of the printing press(D) the development of the Internet(E) the invention of radio

5. Each of the following is mentioned in the passage assomething that the use of language does EXCEPT:

(A) ma king possible the planning and organizationof human endeavors

(B) ena bling societies to exert control over thenatural environment

(C) fos tering the development of literature and thear ts

(D) bot h supporting and upsetting traditional waysof life

(E) hel ping humans better achieve their needs andwants

6. The passage suggests that all of the following areelements of civilization EXCEPT:

(A) the coordination of people’s everyday behavior (B) a single global language(C) control over nature(D) urbanization

(E) social complexity

7. By the term “primary communities” (line 36) theauthor probably means

(A) local groups dependent on direct verbalcommunication

(B) any groups of individuals speaki ng the samelanguage

(C) voluntary associations for the ex change of goods and services

(D) the cities at the core of particula r urbancivilizations

(E) the first urban civilizations to de velop

GO ON TO TH E NEXT PAGE.

33 -25-333

Page 28: LSAT India 2014 Sample Question Paper 4

8/13/2019 LSAT India 2014 Sample Question Paper 4

http://slidepdf.com/reader/full/lsat-india-2014-sample-question-paper-4 28/45

The following are two essays on the daguerreotype,a precursor of photography, in which images are produced on a silver-covered copper plate.

Passage AThough daguerreotypy and photography are

obviously more closely related than daguerreotypy

and painting, and though we do speak of themtogether, it is not mere pedantry to insist that the(5) daguerreotype is not a photograph. The

daguerreotype, like any art form, demands its owncritical vocabulary, its own way of being seen andappreciat ed. For one thing, it had many inherentrestrictio ns that would not have affected the painter or

(10) photogra pher.The daguerreotypist had to arrange a composition

that woul d retain its authority when reduced to about3 inches ( 7.62 cm) square, the most common plate size,and had t o understand the nature of light and shadow

(15) on this hi ghly polished silver plate at variousdistances from a lens. The length of the exposure in

the dague rreotype necessitated the sitter’s being quitestill for s ome minutes, though this was eventuallyreduced t o many seconds. Metal clamps were

(20) invented to hold the sitter’s pose, and children wereoften tied in place; a stiff unnatural pose is one of themost com mon defects found in daguerreotypes, the blurred c hild another. The daguerreotypist had to relyon lightin g, posing, composition, and a personal rapport

(25) with the s itter in order to overcome these potential problems . Still, those who witnessed the beauty of theform lam ented its passing.

Passage BIn March 1839, Samuel Morse wrote that

Daguerre ’s own daguerreotypes were “Rembrandt(30) perfected .” He understood what Daguerre had

wrought, which was a beautiful leap, not of science, but of tec hnology; here, for the first time, was artimmacula tely born of chemistry. The beauty of thedaguerre otype was a compound of awe and delight.

(35) Other wri ters expressed their astonishment in termsthat were quite uniform: “enchanting,” “exquisitely perfect,” “a piece of fairy work.” Such fancy rhetoricis the exp ression of the unique thrill of seeing a brilliant a nd novel art form for the first time. And yet

(40) a great de al of this original pleasure remains for ustoday.

Here in my hand is a daguerreotype. It is notfamous, and it cannot be reproduced. The plate, as if it were a jewel, is enclosed in a case stamped with

(45) scrollwork. The image inside is the size of one’s palm.Unlike any but the smallest paintings, it is an intimateart, meant only for one person at a time. Unfoldingthe case, one has on the left a rectangle of purplevelvet, bordered with embossed gold leaves. The

(50) right-hand panel, seen straight on, is baffling: here isa small mirror in which one sees only part of one’sface, superimposed onto three ghostlike shapes, gray-negative faces with blank eyes atop the shape of somesort of clothing. But turn the mirror slightly and three

(55) full characters blaze out at you—a woman, man, andchild. They are not merely an amalgam of mercuryand silver on a copper surface, but a trace of lifefrozen forever in the sharpest detail of light andshadow.

8. Each passage provides information sufficient to answer which one of the following questions?

(A) Roughly how much did it cost to produce adaguerreotype?

(B) What prevented the widespread use of daguerreotypes for purposes oth er than portraiture?

(C) Roughly how large was the typi caldaguerreotype?

(D) When did photography on paper supersededaguerreotypy?

(E) What were some words used by viewers uponexperiencing daguerreotypes fo r the first time?

9. Passage A, but not passage B, argues tha t(A) the daguerreotype is not a photo graph(B) the daguerreotype is an exquisite art form(C) the daguerreotype is the most pr ecise form of

pictorial art(D) daguerreotypes are commonplac e and usually

imperfect(E) daguerreotypes are complex, ela borate, and

expensive

10. Which one of the following most accura tely describeshow passage B relates to the statement i n passage Athat the daguerreotype demands its own way of being

seen and appreciated?(A) Passage B was written as a direc t response to

this statement.(B) Passage B provides information that could be

used to undermine this stateme nt.(C) Passage B presents an attitude s uggesting

disagreement with this statemen t.(D) Passage B provides an example that illustrates

this statement.(E) Passage B provides the historica l context

necessary to understand this sta tement.

GO ON TO THE NEXT PAGE.

33 -26- 333

Page 29: LSAT India 2014 Sample Question Paper 4

8/13/2019 LSAT India 2014 Sample Question Paper 4

http://slidepdf.com/reader/full/lsat-india-2014-sample-question-paper-4 29/45

11. Which one of the following pairs of lectures is mostanalogous to passage A and passage B in terms of howthese two passages stand in relation to each other?

(A) Lecture 1 includes an evaluation of a particular pantomime performance; lecture 2 includes anargument regarding the ways in which

pantomime may have influenced other artforms.

(B) Lecture 1 includes an argument that pantomimeis truly an art form; lecture 2 includes aclassification of different types of pantomime performances.

(C) Lec ture 1 includes a discussion of how pantomime developed out of other related performing arts; lecture 2 includes informationabout how pantomime has been discussed inliterature.

(D) Lec ture 1 includes a conjecture about why pantomime is no longer performed very often;lecture 2 includes an argument that pantomimists rarely achieve high levels of skillin their art.

(E) Lec ture 1 includes information about thedifficulties that pantomimists face in practicingtheir art; lecture 2 includes a description of a particular pantomime performance.

12. It can be inferred that the authors of both passages believe which one of the following?

(A) The daguerreotype should not be considered atype of photograph.

(B) There is no compelling reason for daguerreotypy not to be practiced today.

(C) Though there are striking exceptions, mostdaguerreotypes are aesthetically unremarkable.

(D) Daguerreotypy was not useful for creatinglandscapes.

(E) The demise of daguerreotypy represents asignificant aesthetic loss.

GO ON TO TH E NEXT PAGE.

33 -27-333

Page 30: LSAT India 2014 Sample Question Paper 4

8/13/2019 LSAT India 2014 Sample Question Paper 4

http://slidepdf.com/reader/full/lsat-india-2014-sample-question-paper-4 30/45

A commission appointed by the government of the Canadian province of Ontario offeredrecommendations to assist judges and lawyers inrecognizing language or actions that may cause some

(5) participants to feel excluded and thereforedisenfranchised during court proceedings. One area of focus was the use of courtroom interpreters for peoplewho are not fluent speakers of English or French.Although the Supreme Court of Canada had given

(10) explicit direction in 1994 concerning an accused person’s right to interpreter assistance, many participa nts in the Canadian legal system still hadconcerns about the use of interpreters. In response tothese con cerns, the commission emphasized that all

(15) those inv olved in proceedings must understand therole of th e interpreter. With this objective in mind, thecommissi on stressed that all parties involved in legal proceedin gs must be made aware of the nature of interpreta tion.

(20) In or der for judges and lawyers to make effectiveuse of an interpreter in the courtroom, they mustunderstan d when an interpreter is necessary,appreciat e the time required for interpretation, anddevelop a n awareness of the nature of culturally

(25) informed interpretation as contrasted with mere literalinterpreta tion. For example, uninformed judges andlawyers o ften expect interpreters to translate what issaid word for word. In practice, however, this type of translatio n frequently fails to convey culturally

(30) specific meaning accurately and effectively, and issometime s simply impossible because each languageis structu rally unique. One interpreter interviewed bythe comm ission explained that while one languagemay use a word or short phrase to express a particular

(35) idea, othe rs have no similarly concise equivalent,requiring the interpreter to use long descriptions of ideas in o ne language that can be expressed briefly inanother. Many interpreters find that in the courtroom,uninform ed judges and lawyers may suspect an

(40) interprete r of embellishing if the interpreter takes along time to explain a point.

Can adian law insists on impartiality ininterpreta tion services. Parties to proceedings,relatives and friends of such parties, or persons

(45) otherwise close to the events giving rise to anaccusatio n are ordinarily viewed as inappropriateinterprete rs in criminal proceedings. However, because some linguistic-minority communities, such

as aboriginal communities, are small, in practice,(50) court participants often know the court interpreter. Inmany cases, prior acquaintance does not matter andmay be unavoidable. But to ensure fair proceedings, adefendant, victim, or other witness must fullyunderstand the interpreter’s role and be able to object

(55) to an interpreter whom he or she does not trust to beimpartial. To these ends, the commissionrecommended that judges make clear in open courtthat the interpreter is a neutral professional, employed by the court to translate what is being said; that a

(60) defendant or any witness may object to a potentially biased interpreter; and that a defendant or aninterpreter may request clarification at any time.

13. Which one of the following most accurately states themain point of the passage?

(A) According to a commission in Ontario, theaccuracy of courtroom translations can never really be guaranteed, and so judicial fairnesscan only be ensured by assigning this role to

professional interpreters.(B) A commission in Ontario has found that

impartiality in interpretation services is notfully achievable in certain kinds of communities,so it is vital that all participants in courtroom proceedings held in such comm unities be ableto reject interpreters they believ e to be biased.

(C) An Ontario commission has advi sed that inorder to ensure fairness in cour t proceedings,the role of the courtroom interp reter must beclearly defined and all courtroo m participantsmust understand the nature of t hat role.

(D) An Ontario commission recomm ends that because existing judicial guidel ines for the useof courtroom interpreters do no t adequatelyensure the impartiality guarante ed by Canadianlaw for members of small linguis tic communities,these guidelines must be amplif ied so as toaccommodate the needs of such communities.

(E) An Ontario commission has dete rmined thatensuring fair and equitable trea tment for allcitizens in all judicial proceedin gs is impossibleunless interpreters are neutral p rofessionalswith no bias for or against any of the peopleinvolved in the case.

14. According to the passage, the inadequac y of

word-for-word translations in legal proce edingsinvolving interpreters

(A) is a limitation of which some la wyers and judges are unaware

(B) is less noticeable to interpreters than to other participants in legal proceeding s

(C) makes some courtroom participa nts feeldisenfranchised

(D) makes fair and impartial trials im possible toachieve

(E) results in needlessly lengthy em bellishments onthe part of interpreters

GO ON TO THE NEXT PAGE.

33 -28- 333

Page 31: LSAT India 2014 Sample Question Paper 4

8/13/2019 LSAT India 2014 Sample Question Paper 4

http://slidepdf.com/reader/full/lsat-india-2014-sample-question-paper-4 31/45

15. By “inappropriate interpreters” (lines 46–47) the author most likely is referring to interpreters

(A) who tend to elicit the suspicion that they areembellishing rather than merely interpreting

(B) without previous experience or training ascourtroom translators

(C) who have little understanding of the Canadianlegal system

(D) who are non-native speakers of the languagethey interpret

(E) with a lesser likelihood of being impartial thanis ordinarily regarded as desirable

16. With which one of the following statements concerningimpartiality in interpretation services would the author be most lik ely to agree?

(A) In court cases involving members of smalllinguistic communities, certain factors that areconducive to accurate and effective

interpretation also make complete impartialityof the interpreter more difficult to ensure.(B) The recognition of the need for impartiality in

interpretation services was the primary reasonfor the formation of the Ontario commission.

(C) Bec ause skill in providing a truly accuratetranslation is dependent on a sensitiveawareness of the original speaker’s culture, thegoals of impartiality and accuracy intranslation are irreconcilable.

(D) The need for impartiality in translation servicesis more likely to be satisfied by an alterationof the attitudes and beliefs of judges andlawyers than by an alteration of courtroom pr ocedure.

(E) On ly if witnesses and defendants are free toap point their own courtroom interpreters canthe need for impartiality in interpretationservices be satisfied.

17. According to the passage, the situation in which acourtroom interpreter is acquainted with other participants in courtroom proceedings

(A) is more of a problem in civil cases than incr iminal cases

(B) constitutes evidence that the Ontariocommission’s recommendations are inadequate

(C) does not always affect the fairness of the proceedings negatively

(D) is inconsistent with standards of impartialitydemanded by Canadian law

(E) often leads interpreters to embellish thetestimony they are translating

18. The author mentions the fact that ideas expressedconcisely in one language may take much longer toexpress in another language (lines 32–38) primarily inorder to

(A) indicate why some judges and lawyers mayharbor doubts about the accuracy of certain

courtroom translations(B) emphasize why translations in a judicial context

cannot convey the culturally specific meaningof the testimony being interpreted

(C) stress the lack of awareness of cultures andlanguages other than English an d French onthe part of many judges and la wyers

(D) illustrate why it is often difficult to find acompetent interpreter

(E) explain why an interpreter’s tran slation of courtroom testimony may be co nsiderably briefer than the original testimo ny

19. The passage most strongly suggests whic h one of thefollowing about the role of courtroom in terpreters?

(A) The importance of this role is un derestimated by most judges and lawyers.

(B) A precise understanding of this r ole is onlylikely to be useful to people in smallcommunities.

(C) This role can never be occupied by someonewho is personally acquainted w ith the participants in courtroom proce edings.

(D) The person playing this role is m ost likely toachieve his or her legal purpose if everyoneinvolved understands the nature of the role.

(E) This role cannot be satisfactorily filled bysomeone who is unaware of the Ontariocommission’s definition of the role.

GO ON TO TH E NEXT PAGE.

33 -29-333

Page 32: LSAT India 2014 Sample Question Paper 4

8/13/2019 LSAT India 2014 Sample Question Paper 4

http://slidepdf.com/reader/full/lsat-india-2014-sample-question-paper-4 32/45

For biologists, the term “eye” describes any light-sensitive organ consisting of more than one cell.Although most animals have eyes, eye structures varywidely. The compound eyes of insects and other

(5) arthropods, for example, have an architecturestrikingly different from the single-lens eyes of vertebrates and mollusks. Until recently, most biologists believed that all the different kinds of eyesevolved independently from as many as forty

(10) ancestral prototypes, and not from a single ancestral prototype eye.

Trad itional means of tracking the evolutionarydevelopm ent of eyes included examinations of internal e ye structures, which tended to support the

(15) multiple origin theory despite some anomalies such asthe resem blance between mammals’ eyes and the eyesof the na utilus mollusk—animals that are not closelyrelated. P roponents of the multiple origin theorydismissed such examples as textbook cases of

(20) evolution ary convergence: the idea that evenstrikingly different prototype eyes could evolve intokinds of e yes remarkably similar to each other.

In su pport of their theory, these biologists point tothe fact t hat different species inhabiting the same

(25) environm ent frequently have very different eyestructures from one another. This lack of correlation between eye structures and physical environments has provided what is perhaps the strongest support for themultiple origin theory. Adherents of the theory argue

(30) that if ey es originated from a single ancestral prototype , then there should be similarity in the patterns o f eye evolution in species that evolved under the same environmental conditions. The inability of biologists to identify such evolutionary patterns is

(35) arguably the primary reason for this theory’swidespre ad acceptance.

In 1993, however, a crucial link was discovered:a control gene that activates the many genes neededfor comp lete eye formation in fruit flies. Analogues to

(40) this gene have since been identified in manyorganism s, including earthworms, mice, and humans,and are e xpected to exist in all eye-bearing organisms.Research ers discovered that inserting the control gene present in mice into fruit flies results in the formation

(45) of functio nal fruit fly eyes. This suggests that thecontrol g enes in mice and in fruit flies areinterchan geable and hence evolved from a single,common ancestral gene. The same may be true of all

of these control genes, which would argue(50) convincingly against the multiple origin theory andcall for a reevaluation of the evidence that seemed tosupport the theory. For example, the lack of correlation between eye structures and physicalenvironments may have resulted from the advantage

(55) bestowed by eyes. The survival value of sight is perhaps so great that even variations in eyes thatmight be less than optimal for some individuals in a particular environment are sufficiently advantageousthat they allow the individuals to survive and

(60) propagate the variation, thus facilitating the proliferation of variations in eyes even in the absenceof an environmental difference.

20. The author mentions that biologists accepted which oneof the following as evidence for the theory that eyesevolved from multiple origins?

(A) the lack of fossil evidence of a commonancestor for all eye-bearing species

(B) the lack of correlation between eye structures

and physical environments(C) the lack of correlation between physical

environments and control genes for eyedevelopment

(D) the resemblance between the eyes of mammalsand the nautilus mollusk, speci es that are notclosely related

(E) the obvious evolutionary advant age bestowed bysome kinds of eyes as compare d with others

21. The primary purpose of the passage is to

(A) distinguish between two theories and explainthe theoretical basis for each of those theories

(B) argue that a particular discovery providesinsufficient evidence for the rej ection of a particular widely accepted theo ry

(C) explain how a particular piece o f evidencechallenges a particular theory t hat has beenwidely accepted

(D) provide grounds for a reexamina tion of theassumptions underlying a recen t challenge toa commonly held theory

(E) suggest some practical implicati ons of a particular theoretical finding th at conflictswith a particular commonly hel d theory

22. The passage provides the strongest supp ort for theinference that the author’s attitude regardi ng the discoveryof a control gene responsible for activati ng eyeformation in fruit flies can be accurately described as

(A) concern that biologists have acce pted the singleorigin theory without carefully evaluating theassumptions it is based on

(B) concern that biologists may have prematurelyabandoned the multiple origin t heory

(C) confidence that the discovery wi ll imminentlylead to the complete abandonm ent of themultiple origin theory

(D) optimism that its discovery might foster areevaluation of the assumptions biologists

utilize when researching evolution in general(E) anticipation that the discovery will foster further

efforts to determine whether all varieties of eyeshave evolved from a single ancestral prototype eye

GO ON TO THE NEXT PAGE.

33 -30- 333

Page 33: LSAT India 2014 Sample Question Paper 4

8/13/2019 LSAT India 2014 Sample Question Paper 4

http://slidepdf.com/reader/full/lsat-india-2014-sample-question-paper-4 33/45

23. According to the author, control genes have been found that

(A) determine how each species’ characteristic eyestructure will differ from that of other species

(B) probably occur in all animals but only activateeye formation in just a few

(C) may be interchangeable among mice, fruit flies, and

humans without causing members of those speciesto develop eyes that are atypical for their species

(D) regulate specific aspects of eye evolution incertain species, suggesting that those species neednot be assumed to stem from a common ancestor

(E) acti vate eye formation in species includinghumans and mice, and this helps explain howevolutionary convergence occurs

24. Proponents of the multiple origin theory would likelyregard the relationship between the development of human eyes and the development of the eyes of thenautilus mollusk as most analogous to which one of the following?

(A) Traditional Italian pizza is very similar in form

and ingredients to a traditional Southern Frenchfood, which, like Italian pizza, has its originsin earlier Mediterranean cultures.

(B) Mexican traditional cuisine uses round, flattortillas made from corn or wheat, andtraditional cuisines of India use tortilla-likeflat, round bread made from wh eat flour, eventhough the Mexican and Indian cultures haveno traditional connection with e ach other.

(C) Yogurt is superficially unlike ch eese, although both are made almost entirely o f milk and bothare traditional ingredients in M iddle Easternand European cuisines.

(D) Mozzarella cheese is traditionall y made fromcow’s milk in the United States , even thoughItalian mozzarella, which is the original modelfor the US version and has alm ost the sameflavor, color, and texture, is tra ditionally madefrom buffalo’s milk.

(E) The culinary use of corn—a veg etable that wasoriginally available only to vari ous NativeAmerican cultures—has spread t o many culturesthroughout the world, some of which are verydistant and different from one a nother.

S T O PIF YOU FIN ISH BEFORE TIME IS CALLED, YOU MAY CHECK YOUR WORK ON THIS SECT ION ONLY.DO NOT WORK ON ANY OTHER SECTION IN THE TEST.

33 -31-333

Page 34: LSAT India 2014 Sample Question Paper 4

8/13/2019 LSAT India 2014 Sample Question Paper 4

http://slidepdf.com/reader/full/lsat-india-2014-sample-question-paper-4 34/45

44 -32- 4 4 4 4SECTION IV

Time—35 minutes

25 Questions

Directions: The questions in this section are based on the reasoning contained in brief statements or passages. For somequestions, more than one of the choices could conceivably answer the question. However, you are to choose the best answer; that

is, the response that most accurately and completely answers the question. You should not make assumptions that are bycommonsense standards implausible, superfluous, or incompatible with the passage. After you have chosen the best answer, blacken the corresponding space on your answer sheet.

1. Journalist: Many people working on difficult prob lems in mathematics report going to sleepwith out a solution, but upon awaking discover they have a solution in mind. This phenomenonoccu rs among all age groups past infancy.

Which one of the following is most strongly supported by the jour nalist’s statements?

(A) Eve ryone’s mind works unconsciously ondifficult problems while sleeping.

(B) Cer tain types of mathematical problems cannot be solved while one is consciously seeking asolution.

(C) Bei ng well rested is a necessary condition for finding a solution to a difficult problem.

(D) Co nsciously seeking a solution is not the onlymental process by which one can solve amathematical problem.

(E) The ability to carry out mental processinggr adually develops during infancy.

2. Over the p ast few decades dozens of people haveclaimed to have sighted the Yeti in the Himalayas. This

provides st rong evidence that the creature exists.The reason ing in the argument is questionable becausethe argume nt fails to

(A) tak e into account similar sightings in mountainsother than the Himalayas

(B) con sider alternative explanations for the reportedsightings

(C) con sider the absence of photographs of the Yeti(D) eva luate historical evidence for the existence of

the Yeti(E) acc ount for why most people still do not believe

in the Yeti

3. Migraines are caused by changes in certain blood vesselsof the brain that result in a local disturba nce in theaction of a specific nerve-activating che mical. Twomigraine remedies, drug K and drug L, h ave exactlythe same restorative effects on the local action of thischemical, but L also disrupts the action of several other chemicals in the brain that govern menta l activity andthe body’s metabolism.

The information above most strongly su pports which

one of the following hypotheses?(A) Treatment with L is likely to be slower-acting

in its effect on a migraine than is treatmentwith K.

(B) Neither treatment with L nor tre atment with K is likely to have a significant ef fect on thesymptoms of a migraine.

(C) People whose migraines are trea ted with L aremore likely to experience relief from pain thanare people whose migraines are treated with K.

(D) People whose migraines are trea ted with L arelikely to experience a wider ran ge of effects beyond the cessation of migrain es than are

people whose migraines are tre ated with K.(E) K, unlike L, should be administe red immediatelyafter the onset of a migraine.

GO ON TO TH E NEXT PAGE.

Page 35: LSAT India 2014 Sample Question Paper 4

8/13/2019 LSAT India 2014 Sample Question Paper 4

http://slidepdf.com/reader/full/lsat-india-2014-sample-question-paper-4 35/45

4. Advertisement: The new Reflex computer represents aconceptual advance. Unlike traditional computers,the Reflex has a built-in monitoring function thatcontinuously checks all other computer operationsand lets you know if they are malfunctioning inany way, thus preventing the loss of data. Withthe Reflex, therefore, you’ll never lose dataagain!

Which one of the following is an assumption on whichthe advertisement’s argument depends?

(A) The monitoring function can always determinethe cause of the Reflex’s malfunctioning.

(B) The Reflex computer continues to process datawhile it is warning of a malfunction.

(C) The monitoring function provides suggestionson how to prevent future malfunctions.

(D) The monitoring function of the Reflex does notreduce the computer’s speed.

(E) The monitoring function of the Reflex is not

su bject to frequent unpredictablemalfunctioning.

5. Editorialis t: Research reveals that patients of hospitalcard iac units in which doctors perform manyaggr essive, high-tech procedures are less likely to be alive a month after leaving the hospital thanare patients of hospital cardiac units that relyalm ost exclusively on standard treatments. Thisindi cates that aggressive, high-tech treatments of card iac disease are less effective than standardtreat ments.

Which one of the following, if true, most weakens the

editorialist’ s argument?(A) So me cardiac patients have lived for many years

after receiving aggressive, high-tech treatments.(B) Ho spitals that have high rates of long-term

survival for cardiac patients do not alwayshave high rates of long-term survival for other patients.

(C) All cardiac patients have access to hospitals inwhich relatively large numbers of aggressive,high-tech procedures are performed.

(D) Pat ients with the most-severe cardiac problemstend to receive treatment at hospitals wheremany aggressive, high-tech treatments are

performed.(E) Doctors who do not perform aggressive, high-tech procedures tend to place much emphasison the prevention of cardiac problems throughhealthy lifestyle choices.

6. Economist: When people save their money instead of spending it, that money is not being used to makethe purchases of goods and services that help businesses remain profitable. Because these purchases are needed to generate tax revenuesthat can be used for government-funded scientificresearch, it is clear that industrial growth, whichrelies on the application of this government-funded research, will falter if the global politicalclimate begins to make people anxious or cautious.

Which one of the following is an assum ption required by the economist’s argument?

(A) People become less inclined to s pend their money on goods and services w hen the global political climate makes them an xious or cautious.

(B) The purpose of scientific researc h that is funded by tax money is to lend itself t o applications

that will sustain industrial grow th.(C) People are often made anxious a nd cautious by

the global political climate.(D) The scientific research that is cu rrently funded

by the government will no long er take place if tax revenues decrease.

(E) People who are more inclined to spend than tosave their money are neither ca utious nor anxious.

GO ON TO TH E NEXT PAGE.

44 -33-4 4 4 4

Page 36: LSAT India 2014 Sample Question Paper 4

8/13/2019 LSAT India 2014 Sample Question Paper 4

http://slidepdf.com/reader/full/lsat-india-2014-sample-question-paper-4 36/45

7. Until recently, endosulfan was one of the most widelyused pesticides. Some others—toxaphene, dieldrin, andchlordane—were banned or restricted in many countriesin the 1980s but linger in the environment. All four have a weak effect individually in increasing estrogenlevels in women. Scientists have found, however, thattheir potential to cause health hazards increasesdramatically when the chemicals are combined. For example, a mixture of endosulfan and dieldrin had160 to 1,600 times more estrogen-boosting potency thaneither chemical did individually. Increased estrogen isassociated with an increased cancer risk in women.

Which one of the following is most strongly supported by the info rmation above?

(A) Pes ticides that boost estrogen levels are moredangerous than others.

(B) An y two pesticides in combination pose greater health risks than do the same pesticidesuncombined.

(C) Bec ause of its hazards when combined withother chemicals, endosulfan is more dangerousthan most other pesticides.

(D) If c ertain pesticides combine in theenvironment, they may constitute greatlyincreased human health hazards.

(E) Ba nning endosulfan worldwide in the 1980swould have had no effect on worldwidecancer rates.

8. Columnist : In a recent article an economist arguesthat corporations have no responsibility to society beyo nd obeying the law and maximizing profitfor shareholders. But in a different article thesam e economist endorses the view thatcorp orations ought to make financialcont ributions to social programs in thecom munities in which they are located. Thus theecon omist is caught in a contradiction.

The colum nist’s argument is questionable because itfails to rule out the possibility that

(A) cor porations make nonfinancial contributions tosocial programs within their local communities

(B) ma ny shareholders of corporations are in favor of their corporations making contributions tocommunity social programs

(C) social programs have an economic effect on acommunity and are therefore part of thedomain of economists

(D) financial contributions to community social programs improve a corporation’s image in away that improves its profitability

(E) a corporation’s making financial contributions tocommunity social programs violates no laws

9. While conditions on Mars are no longer favorable for the evolution of life, scientists point out that in manyways Mars resembles portions of Antarctica, which domanage to support colonies of microbes. But theseorganisms require liquid water, as do all forms of life.And if there is liquid water on Mars at all, it is onlyseasonal and in small amounts.

The statements above, if true, most strongly supportwhich one of the following?

(A) If there is no life on Mars, then there is noliquid water on Mars.

(B) It is unreasonable to suppose tha t Mars has ever supported any forms of life.

(C) If there are colonies of microbes in Antarctica,then there are colonies of micro bes on Mars.

(D) If there is life on Mars, then the re is liquidwater on Mars.

(E) If there is liquid water on Mars, then there arecolonies of microbes on Mars.

10. Politician: The cohesion of a society dep ends on itsmembers’ accepting a set of basic principles.When these principles are routinel y called intoquestion, the society begins to ero de. Anysociety, therefore, that allows scho ols to presentthe society’s set of basic principle s as simply oneof several alternatives, with the su ggestion thatstudents may choose which to acc ept on the basisof the principles’ worth, is invitin g its owndemise.

Which one of the following would, if tru e, moststrengthen the politician’s argument?

(A) Given the chance to assess the m erits of the principles governing their socie ties, individualswill rarely find those principles acceptable.

(B) One cannot evaluate a set of pol itical principleswithout having first established criteria of evaluation.

(C) Some flourishing societies do no t encouragequestioning of their most basic principles.

(D) Children are more likely than ad ults to questionthe wisdom of those who found ed and shapeda society.

(E) Unless people believe that they have freelychosen to adopt the principles that govern their

societies, they will tend to repudiate these principles.

GO ON TO THE NEXT PAGE.

44 -34- 4 4 4 4

Page 37: LSAT India 2014 Sample Question Paper 4

8/13/2019 LSAT India 2014 Sample Question Paper 4

http://slidepdf.com/reader/full/lsat-india-2014-sample-question-paper-4 37/45

11. Economist: In general, several apprentices workingtogether produce about the same amount in anhour as a single more highly trained worker.Hence the more highly trained worker canusually command several times the hourly wageof an apprentice. Thus if the apprentice wage isincreased, the hourly wages of more highlytrained workers will generally rise by a proportionate amount. Therefore the reason thatmore highly trained workers favor an increasedapprentice wage is that it would increase their own wages.

The econo mist’s reasoning is flawed because theeconomist t akes for granted that

(A) if a policy change that people support wouldwork in their favor, that is why they support it

(B) if o ne event causes another event, then the firstevent occurs whenever the second event occurs

(C) an increase in the apprentice wage would result

in an increase in the wages of all better-paidworkers

(D) the wages of highly trained workers will usuallynot increase unless the apprentice wageincreases

(E) an increase in the apprentice wage would benefitonly highly trained workers

12. Oxygenate d petrol, although it reduces pollution,causes freq uent stalling in poorly maintainedautomobile s. However, in laboratory tests of automobile s that had been driven 100,000 kilometers,those that h ad regularly used oxygenated petrol stalledless than th ose that had regularly used nonoxygenated petrol.

Which one of the following, if true, most helps toexplain the results of the laboratory tests describedabove?

(A) The adverse effects from oxygenated petrol can be distinguished from mechanical engine pr oblems in the laboratory tests.

(B) Au tomobile owners who pay a high price for oxygenated petrol have less money for automobile maintenance than do automobileowners who buy the less expensivenonoxygenated petrol.

(C) Drivers whose automobiles regularly exhibitadverse effects from oxygenated petrolgenerally cease to notice the adverse effects bythe time their automobiles have been driven100,000 kilometers.

(D) In conducting the tests, the laboratory used eachtype of petrol on each type of engine.

(E) Automobile owners who regularly useoxygenated petrol get more frequent enginemaintenance because of the adverse effectsfrom the petrol.

13. It has been hypothesized that dinosaurs became extinctsimply because they grew too large and slow tofunction effectively as predators, but this cannot beright. If that hypothesis were correct, then smalldinosaurs would have survived even though their largecounterparts died off. But it is an undisputed fact thatall dinosaurs—even very small ones—became extinct.

The argument above proceeds by

(A) stating a hypothesis and then supporting it bymeans of an example

(B) introducing a hypothesis and the n questioningthe accuracy of the data on whi ch it is based

(C) refuting a hypothesis by showin g that one of theconsequences suggested by that hypothesis isfalse

(D) disputing a hypothesis by offerin g a plausiblecompeting explanation

(E) refuting a hypothesis by showin g that it isimpossible to produce evidence to the contrary

14. If, when the twenty-third century arrives , the history of the Mughal Empire is better known than that of our time, it will be because of our enthusias m for electronically stored digital files. The co ntents of mostdigital media vanish long before words written on high-quality paper would, and they become o bsolete andunusable even sooner due to rapid techn ologicalinnovation. While information written on paper can beread directly, access to digital informatio n is doublyindirect: the sequence of digits represent ing theinformation must be retrieved, and then t hat sequencemust be decoded by the appropriate soft ware.

Which one of the following statements m ost accuratelyexpresses the main conclusion of the arg ument?

(A) The obsolescence brought about by ongoingtechnological innovation will m ake historicalresearch increasingly difficult i n the future.

(B) Information written on paper is more readilyaccessible than the contents of digitaldocuments.

(C) Historically important records fr om the presentera may be lost because of the medium inwhich they are stored.

(D) Digitally stored information is p articularlyvulnerable because of the two-step process

required to retrieve it.(E) Historians in the future may know more aboutthe Mughal Empire than about the twenty-firstcentury.

GO ON TO THE NEXT PAGE.

44 -35-4 4 4 4

Page 38: LSAT India 2014 Sample Question Paper 4

8/13/2019 LSAT India 2014 Sample Question Paper 4

http://slidepdf.com/reader/full/lsat-india-2014-sample-question-paper-4 38/45

15. Patel: It is often thought that direct experience,unlike language and culture, can always serve ascommon ground for communication amonghuman beings. But people from different culturesinhabit different sensory worlds. Becauseselective screening of sensory data admits some perceptions while f iltering out others, one person’s experience, perceived through one set of culturally patterned sensory screens, is quitedifferent from what anyone from another culturewould experience when encountering the samesens ory data.

Which one of the following is an assumption required by Patel’s a rgument?

(A) No two people from different cultures have thesame sensory screens.

(B) No two people from different cultures ever encounter the same sensory data.

(C) Tw o people who have different perceptions of

the same event must be from different cultures.(D) Tw o people from the same culture sometimes

have different sensory screens.(E) The experience of one person is quite different

fr om the experience of any other person.

16. If the count ry’s income taxes are decreased, the country’seconomy w ill not be strengthened, because many of thecountry’s p ublic employees would become unemployed,thereby fur ther weakening the country’s economy.

The pattern of reasoning in the argument above is mostsimilar to t hat in which one of the followingarguments?

(A) Tax incentives will lure new businesses to theregion, thereby increasing the region’sem ployment, but even if no tax incentives areof fered, employment in the region is likely toincrease.

(B) A s light increase in employment will notincrease confidence in the current government, because people are aware that any increase inem ployment is likely to be temporary.

(C) An increase in interest rates will not increase thenumber of jobs, because increased interest rateswill put many companies out of business, andthis result will decrease the number of jobs.

(D) If both government spending and income taxesare decreased, growth in private businessesmight occur, because people will have moremoney to invest, but decreases in bothspending and taxes are unlikely.

(E) A decrease in taxes on businesses willstrengthen the economy because it willencourage businesses to expand employmentopportunities, even though many workers willnot qualify for the new opportunities.

17. Historian: The only evidence we have for claimsabout the past exists in the present. How thingsactually occurred is beyond our knowledge.Historians construct coherent stories that explainthe available evidence and why the present is asit is. Such stories about the past, however, do notneed to be true to be good history; they needonly explain the evidence about the past andwhat we know about the present better than dorival accounts.

Which one of the following judgments conforms mostclosely to the historian’s position?

(A) It is likely that the 1857 War of Independencedid not actually occur in the wa y we think it did.

(B) It is not necessary to know what actuallyoccurred during the 1857 War of Independenceto write a good history of it.

(C) An account of the 1857 War of Independence

that is true is better as history t han one that isnot true but better explains the evidence.

(D) An account that helps explain th e availableevidence concerning the 1857 War of Independence is no better than one that helpsexplain why the present is as it is.

(E) It is not possible to write a good history of the1857 War of Independence wit hout studyingrival accounts.

GO ON TO TH E NEXT PAGE.

44 -36- 4 4 4 4

Page 39: LSAT India 2014 Sample Question Paper 4

8/13/2019 LSAT India 2014 Sample Question Paper 4

http://slidepdf.com/reader/full/lsat-india-2014-sample-question-paper-4 39/45

18. University administrator: Saying that writing cannot betaught is as silly as saying that playing the violincannot be taught. Of course writing can be taught.Writing classes have been taught at this universityever since it was founded.

The reasoning in the university administrator’s argument

is flawed in that the argument(A) relies on using a key term in two different senses(B) rests entirely on a weak analogy(C) generalizes on the basis of mere speculation(D) trea ts a view with contempt instead of offering

evidence against it(E) tak es for granted that those who can teach one

thing can teach another

19. Any ornam ental garden has both plants and structuralelements su ch as rocks and fences. Because the plantsconstantly change with growth and decay, it is difficultto keep a p roper balance in the garden between the

plants and the structures. Balance can be achieved onlywith carefu l planning, and even when established, balance has to be constantly maintained by pruning andthinning.

Which one of the following most accurately expressesthe main co nclusion of the argument?

(A) It i s difficult to keep a proper balance in agarden between structures and plants.

(B) The proper balance in a garden betweenstr uctures and plants is best achieved andmaintained by careful planning.

(C) The reason why it is difficult to keep a proper balance in a garden between structures and plants is that the plants constantly change withgr owth and decay.

(D) It i s difficult to constantly maintain the balancein a garden between structures and plants by pr uning and thinning.

(E) Wit hout careful planning and maintenance, the balance in a garden between structures and plants constantly changes with growth anddecay.

20. A dog who is emotionally indifferent and not securelyattached to its human companion neither whimperswhen the human leaves the room nor looks up toacknowledge the human’s return. Some dogs do notwhimper when their human companions leave them atkennels. These dogs, therefore, are emotionallyindifferent and not securely attached to their humancompanions.

The flawed pattern of reasoning in the argument aboveis most similar to that in which one of the following?

(A) A happy person is neither bitter nor depressed.Some successful people are bitt er. These people, therefore, are not happy .

(B) Creative people do not fear atte mpting to solvenew problems and do not confin e their thinkingto solutions other people propo se. Neena is notafraid of trying to solve new pr oblems.Therefore, Neena is a creative person.

(C) If a dolphin is well cared for, th en it is easily

trained. Some dolphins are not well cared for.These dolphins, therefore, are n ot easilytrained.

(D) At the bookstore’s anniversary s ale all novelsare being sold at a discount. Th is travel guideis being sold at the bookstore b ut not at adiscount. Therefore, the bookst ore’sanniversary sale has not begun yet.

(E) If new jobs are created, the une mployment ratewill decrease. New jobs are bei ng created by both new industries and rejuven ated older industries. Therefore, the unem ployment ratewill decrease.

21. Old rose varieties have less vibrant color s than modernrose varieties, but are superior in fragran ce. Thissituation has arisen because the modern rose varietieswere developed to meet customer deman d andmarketed mainly through catalogs: color s, but notfragrances, can be photographed.

Which one of the following, if true, mos t strengthensthe support for the causal claim made ab ove?

(A) People in earlier times were inte rested in thefragrances of roses as well as t heir colors.

(B) Breeding roses to enhance one tr ait often hasunanticipated consequences for other traits.

(C) Old rose varieties are becoming increasingly popular as people discover their fragrances.(D) People in earlier times would have found the

vibrant colors of modern roses too harsh.(E) State-of-the-art photographic equipment makes

possible unprecedentedly accurate color reproductions of roses.

GO ON TO THE NEXT PAGE.

44 -37-4 4 4 4

Page 40: LSAT India 2014 Sample Question Paper 4

8/13/2019 LSAT India 2014 Sample Question Paper 4

http://slidepdf.com/reader/full/lsat-india-2014-sample-question-paper-4 40/45

22. Analyst: The increasing complexity of computers maylead those who pursue a career in computer programming to think that job security and higher wages can be attained by becoming morespecialized as the field becomes more complex.Even though specialists earn higher wages thangeneralists within computer programming, thismove is ill-advised because one risks specializingin a technology that will become obsolete.Consider the plight of people who used to repair eight-track tape players.

Which one of the following, if true, most weakens theanalyst’s ar gument?

(A) Ma ny other careers will soon offer greater jobsecurity and higher wages than computer pr ogramming.

(B) So me current computer technologies will not become obsolete within the lifetimes of anyonenow pursuing a career in computer

pr ogramming.(C) The average wages earned by generalists within

computer programming will never be as highas the average wages earned by specialistswithin computer programming.

(D) The current technological knowledge of mostspecialists within computer programming couldalso be applied to many technologies that willre place present ones.

(E) The technological complexity of computers mayeventually grow less rapidly than at present.

23. Any great ape can learn to recognize its reflection in amirror. Any animal that recognizes its reflection in amirror has a sense of self. An animal has a sense of theminds of others only if it possesses a sense of self.

Which one of the following follows logically from theinformation above?

(A) Before a great ape develops a sense of theminds of others, it must learn to recognize itsreflection.

(B) All great apes have the ability to develop asense of the minds of others.

(C) Any animal that does not recogni ze its reflectionin a mirror does not have a sen se of self.

(D) If an animal is not a great ape it is not capableof developing a sense of the mi nds of others.

(E) Any animal that is incapable of developing asense of self is not a great ape.

GO ON TO TH E NEXT PAGE.

44 -38- 4 4 4 4

Page 41: LSAT India 2014 Sample Question Paper 4

8/13/2019 LSAT India 2014 Sample Question Paper 4

http://slidepdf.com/reader/full/lsat-india-2014-sample-question-paper-4 41/45

24. Incumbent politicians tend to win elections in times of economic prosperity and lose during recessions.Knowing this, incumbents have an incentive to boostthe economy temporarily by cutting taxes and raisinggovernment spending several months before anelection. Thus, in democratic nations, upcomingelections may be a major cause of short-term economicexpansions.

Each of the following, if true, strengthens the argumentEXCEPT:

(A) Pol iticians in democratic nations sometimes cuttaxes and raise government spending in themonths following an election.

(B) In democratic nations, short-term economicex pansions more often start within the sixmonths preceding a national election thanwithin the six months following one.

(C) Cut ting taxes several months before an electionis an effective means for incumbent politicians

to boost the economy temporarily.(D) In democratic nations, incumbent politicians

have more control than anyone else over decisions about when taxes will be cut andgovernment spending raised.

(E) Rai sing government spending several months before an election is an effective means for incumbent politicians to boost the economytemporarily.

25. Some educators have had remarkable success with anunusual approach to science education. For example, a physics lesson on the properties of waves might begin by plucking guitar strings. After observing the stringsvibrate, students would begin to apply names to the phenomena they had witnessed, such as frequency andamplitude. Only after fully understanding these real-lifeconcepts would they begin to apply abstract terms andequations to the phenomena.

Which one of the following principles is best illustrated by the example above?

(A) Students will understand abstrac t terms only if they are applicable to phenome na they haveobserved.

(B) Science education is most succe ssful when itemphasizes observation and exp eriment rather than mathematical calculation.

(C) Students are unable to form abst ract conceptsconcerning phenomena until the y become

familiar with these phenomena.(D) Students learn best when teacher s appeal to the

interests of their students.(E) Familiarity with phenomena faci litates the

learning of more formal treatm ents of these phenomena.

S T O PIF YOU FIN ISH BEFORE TIME IS CALLED, YOU MAY CHECK YOUR WORK ON THIS SECT ION ONLY.

DO NOT WORK ON ANY OTHER SECTION IN THE TEST.

44 -39-4 4 4 4

Please remain seated until all test books and answer sheets have been collected and checked.

Page 42: LSAT India 2014 Sample Question Paper 4

8/13/2019 LSAT India 2014 Sample Question Paper 4

http://slidepdf.com/reader/full/lsat-india-2014-sample-question-paper-4 42/45

40

NO TEST MATERIAL ON THIS PAGE.

Page 43: LSAT India 2014 Sample Question Paper 4

8/13/2019 LSAT India 2014 Sample Question Paper 4

http://slidepdf.com/reader/full/lsat-india-2014-sample-question-paper-4 43/45

41

Acknowledgment is made to the following sources from which material has been adapted for use in this test booklet:

Final Report of the Commission on Systemic Racism in the Ontario Criminal Justice System . ©1995by Queen’s Printer for Ontario.

“Harmless Chemicals Yield Dangerous Combinations.” ©1996 by the National Wildlife Federation.

“Space Shuttle Science Shows How 1908 Tunguska Explosion Was Caused By A Comet.” ©2009by ScienceDaily LLC. http://www.sciencedaily.com/releases/2009/06/090624152941.htm.

Page 44: LSAT India 2014 Sample Question Paper 4

8/13/2019 LSAT India 2014 Sample Question Paper 4

http://slidepdf.com/reader/full/lsat-india-2014-sample-question-paper-4 44/45

42

ANSWER KEY

SECTION I

1. D 8. C 15. D2. D 9. D 16. E3. A 10. E 17. D4. D 11. B 18. B5. C 12. E 19. D6. E 13. A 20. B7. B 14. B

SECTION II

1. E 8. C 15. D 22. C2. A 9. E 16. C 23. E3. D 10. D 17. B4. B 11. A 18. C5. D 12. B 19. D6. A 13. E 20. C7. E 14. A 21. D

SECTION III

1. E 8. C 15. E 22. E2. D 9. A 16. A 23. C3. D 10. D 17. C 24. B4. A 11. E 18. A5. C 12. E 19. D6. B 13. C 20. B7. A 14. A 21. C

SECTION IV1. D 8. D 15. A 22. D2. B 9. D 16. C 23. E3. D 10. A 17. B 24. A4. E 11. A 18. A 25. E5. D 12. E 19. A6. A 13. C 20. B7. D 14. C 21. B

Page 45: LSAT India 2014 Sample Question Paper 4

8/13/2019 LSAT India 2014 Sample Question Paper 4

http://slidepdf.com/reader/full/lsat-india-2014-sample-question-paper-4 45/45